Respiratory system exam 7 133

¡Supera tus tareas y exámenes ahora con Quizwiz!

The nurse can best determine adequate arterial oxygenation of the blood by assessing A. Heart rate B. Hemoglobin level C. Arterial oxygen partial pressure. D. Arterial carbon dioxide partial pressure

C. Arterial oxygen partial pressure

Stridor

Continuous musical or crowing sound of constant pitch. Result of partial obstruction of larynx or trachea.

Value-belief

non compliance with treatment plan, conflict with values

Pneumonia

Inspection: Tachypnea, use o accessory muscles, duskiness or cyanosis Palpation: Increase fremitus over affected area Percussion: Dull over affected areas Auscultation: Early: Bronchial sounds Later: Fine and/or coarse crackles, egophoney, whispered pectoriloquy

Pleural effusion

Inspection: Tachypnea, use of accessory usces Palpation: Increase movement, Increase Fremitus above effusion, Absent fremitus over effusion Percussion: Dull Auscultation: Diminished or absent over effusion, egophony above effusion

PEFR (Peak expiratory flow rate)

Maximum rate of airflow durgin forced expiration

Describe Tachypnea

respirations > 35 clinical significance/contributing factors: - respiratory failure - response to fever - anemia - pain - respiratory infection - anxiety (emergencies SNS system kicks in)

Identify initial assessment findings for a patient with EARLY STAGE LEFT sided heart failure

- fatigue - breathlessness - dizziness - confusion as a result of tissue hypoxia from the diminished CO

What should the nurse inspect when assessing a patient with shortness of breath for evidence of long-standing hypoxemia? 1. Chest excursion 2. Spinal curvatures 3. Respiratory pattern 4. Fingernails and their base

2. Fingernails and their base a. Clubbing, a sign of long-standing hypoxemia, is evidenced by an increase in the angle between the base of the nail and the fingernail to 180 degrees or more, usually accompanied by an increase in the depth, bulk, and sponginess of the end of the finger.

When assessing a patient's sleep-rest pattern related to respiratory health, what should the nurse ask the patient about (select all that apply)? 1. Have trouble falling asleep? 2. Need to urinate during the night? 3. Awaken abruptly during the night? 4. Sleep more than 8 hours per night? 5. Need to sleep with the head elevated?

1. Have trouble falling asleep?; Awaken abruptly during the night?; Need to sleep with the head elevated? a. The patient with sleep apnea may have insomnia and/or abrupt awakenings. Patients with cardiovascular disease (e.g., heart failure that may affect respiratory health) may need to sleep with the head elevated on several pillows (orthopnea). Sleeping more than 8 hours per night or needing to urinate during the night is not indicative of impaired respiratory health.

Thoracentesis procedures, these are the actions in order the nurse should complete them in

1. Obtain the supplies that will be used 2. Direct the family members to the waiting room 3. Position the patient sitting upright with the elbows on an over-the-bed table 4. Instruct the patient not to talk during the procedure 5. Observe for signs of hypoxia during the procedure 6. Verify breath sounds in all fields 7. Send labeled specimen containers to the lab

After swallowing, a 73-year-old patient is coughing and has a wet voice. What changes of aging could be contributing to this abnormality? 1. Decreased response to hypercapnia 2. Decreased number of functional alveoli 3. Increased calcification of costal cartilage 4. Decreased respiratory defense mechanisms

10. Decreased respiratory defense mechanisms a. These manifestations are associated with aspiration, which more easily occur in the right lung as the right mainstem bronchus is shorter, wider, and straighter than the left mainstem bronchus. Aspiration occurs more easily in the older patient related to decreased respiratory defense mechanisms (e.g., decreases in immunity, ciliary function, cough force, sensation in pharynx). Changes of aging include a decreased response to hypercapnia, decreased number of functional alveoli, and increased calcification of costal cartilage, but these do not increase the risk of aspiration.

The patient is hospitalized with pneumonia. Which diagnostic test should be used to measure the efficiency of gas transfer in the lung and tissue oxygenation? 1. Thoracentesis 2. Bronchoscopy 3. Arterial blood gases 4. Pulmonary function tests

11. Arterial blood gases a. Arterial blood gases are used to assess the efficiency of gas transfer in the lung and tissue oxygenation as is pulse oximetry. Thoracentesis is used to obtain specimens for diagnostic evaluation, remove pleural fluid, or instill medication into the pleural space. Bronchoscopy is used for diagnostic purposes, to obtain biopsy specimens, and to assess changes resulting from treatment. Pulmonary function tests measure lung volumes and airflow to diagnose pulmonary disease, monitor disease progression, evaluate disability, and evaluate response to bronchodilators.

The nurse, when auscultating the lower lungs of the patient, hears these breath sounds. How should the nurse document these sounds? 1. Stridor 2. Rhonchi 3. Coarse crackles 4. Bronchovesicular

12. Coarse crackles a. Coarse crackles are a series of long-duration, discontinuous, low-pitched sounds caused by air passing through an airway intermittently occluded by mucus, an unstable bronchial wall, or a fold of mucosa. Coarse crackles are evident on inspiration and at times expiration. Stridor is a continuous crowing sound of constant pitch from partial obstruction of larynx or trachea. Rhonchi are a continuous rumbling, snoring, or rattling sound from obstruction of large airways with secretions. Bronchovesicular sounds are normal sounds heard anteriorly over the mainstem bronchi on either side of the sternum and posteriorly between the scapulae with a medium pitch and intensity.

The patient is calling the clinic with a cough. What assessment should be made first before the nurse advises the patient? 1. Cough sound, sputum production, pattern 2. Frequency, a family history, hematemesis 3. Smoking, medications, residence location 4. Weight loss, activity tolerance, orthopnea

13. Cough sound, sputum production, pattern a. The sound of the cough, sputum production and description, as well as pattern of the cough's occurrence (including acute or chronic) and what its occurrence is related to are the first assessments to be made to determine the severity. Frequency of the cough will not provide a lot of information. Family history can help to determine a genetic cause of the cough. Hematemesis is vomiting blood and not as important as hemoptysis. Smoking is an important risk factor for COPD and lung cancer and may cause a cough. Medications may or may not contribute to a cough as does residence location. Weight loss, activity intolerance, and orthopnea may be related to respiratory or cardiac problems, but are not as important when dealing with a cough.

During the assessment in the ED, the nurse is palpating the patient's chest. Which finding is a medical emergency? 1. Trachea moved to the left 2. Increased tactile fremitus 3. Decreased tactile fremitus 4. Diminished chest movement

14. Trachea moved to the left a. Tracheal deviation is a medical emergency when it is caused by a tension pneumothorax. Tactile fremitus increases with pneumonia or pulmonary edema and decreases in pleural effusion or lung hyperinflation. Diminished chest movement occurs with barrel chest, restrictive disease, and neuromuscular disease.

The patient with Parkinson's disease has a pulse oximetry reading of 72%, but he is not displaying any other signs of decreased oxygenation. What is most likely contributing to his low SpO2 level? 1. Motion 2. Anemia 3. Dark skin color 4. Thick acrylic nails

15. Motion a. Motion is the most likely cause of the low SpO2 for this patient with Parkinson's disease. Anemia, dark skin color, and thick acrylic nails as well as low perfusion, bright fluorescent lights, and intravascular dyes may also cause an inaccurate pulse oximetry result. There is no mention of these or reason to suspect these in this question

In assessment of the patient with acute respiratory distress, what should the nurse expect to observe (select all that apply)? 1. Cyanosis 2. Tripod position 3. Kussmaul respirations 4. Accessory muscle use 5. Increased AP diameter

16. Tripod position; Accessory muscle use a. Tripod position and accessory muscle use indicate moderate to severe respiratory distress. Cyanosis may be related to anemia, decreased oxygen transfer in the lungs, or decreased cardiac output. Therefore it is a nonspecific and unreliable indicator of only respiratory distress. Kussmaul respirations occur when the patient is in metabolic acidosis to increase CO2 excretion. Increased AP diameter occurs with lung hyperinflation from COPD, cystic fibrosis, or with advanced age.

The nurse is caring for a patient with chronic obstructive pulmonary disorder (COPD) and pneumonia who has an order for arterial blood gases to be drawn. What is the minimum length of time the nurse should plan to hold pressure on the puncture site? 1. 2 minutes 2. 5 minutes 3. 10 minutes 4. 15 minutes

3. 5 minutes a. After obtaining blood for an arterial blood gas measurement, the nurse should hold pressure on the puncture site for 5 minutes by the clock to be sure that bleeding has stopped. An artery is an elastic vessel under much higher pressure than veins, and significant blood loss or hematoma formation could occur if the time is insufficient.

A patient with a recent history of a dry cough has had a chest x-ray that revealed the presence of nodules. In an effort to determine whether the nodules are malignant or benign, what is the primary care provider likely to order? 1. Thoracentesis 2. Pulmonary angiogram 3. CT scan of the patient's chest 4. Positron emission tomography (PET)

4. Positron emission tomography (PET) a. ET is used to distinguish benign and malignant pulmonary nodules. Because malignant lung cells have an increased uptake of glucose, the PET scan (which uses an IV radioactive glucose preparation) can demonstrate increased uptake of glucose in malignant lung cells. This differentiation cannot be made using CT, a pulmonary angiogram, or thoracentesis.

A patient with recurrent shortness of breath has just had a bronchoscopy. What is a priority nursing action immediately following the procedure? 1. Monitor the patient for laryngeal edema. 2. Assess the patient's level of consciousness. 3. Monitor and manage the patient's level of pain. 4. Assess the patient's heart rate and blood pressure.

5. Monitor the patient for laryngeal edema. a. Priorities for assessment are the patient's airway and breathing, both of which may be compromised after bronchoscopy by laryngeal edema. These assessment parameters supersede the importance of loss of consciousness (LOC), pain, heart rate, and blood pressure, although the nurse should also be assessing these.

What is the normal tidal volume of a patient

500mL

After assisting at the bedside with a thoracentesis, the nurse should continue to assess the patient for signs and symptoms of what? 1. Bronchospasm 2. Pneumothorax 3. Pulmonary edema 4. Respiratory acidosis

6. Pneumothorax a. Because thoracentesis involves the introduction of a catheter into the pleural space, there is a risk of pneumothorax. Thoracentesis does not carry a significant potential for causing bronchospasm, pulmonary edema, or respiratory acidosis.

The patient had abdominal surgery yesterday. Today the lung sounds in the lower lobes have decreased. The nurse knows this could be due to what occurring? 1. Pain 2. Atelectasis 3. Pneumonia 4. Pleural effusion

7. Atelectasis a. Postoperatively there is an increased risk for atelectasis from anesthesia as well as restricted breathing from pain. Without deep breathing to stretch the alveoli, surfactant secretion to hold the alveoli open is not promoted. Pneumonia will occur later after surgery. Pleural effusion occurs because of blockage of lymphatic drainage or an imbalance between intravascular and oncotic fluid pressures, which is not expected in this case.

The patient's arterial blood gas results show the PaO2 at 65 mmHg and the SaO2 at 80%. What early manifestations should the nurse expect to observe in this patient? 1. Restlessness, tachypnea, tachycardia, and diaphoresis 2. Unexplained confusion, dyspnea at rest, hypotension, and diaphoresis 3. Combativeness, retractions with breathing, cyanosis, and decreased output 4. Coma, accessory muscle use, cool and clammy skin, and unexplained fatigue

8. Restlessness, tachypnea, tachycardia, and diaphoresis a. With inadequate oxygenation, early manifestations include restlessness, tachypnea, tachycardia, and diaphoresis, decreased urinary output, and unexplained fatigue. The unexplained confusion, dyspnea at rest, hypotension, and diaphoresis; combativeness, retractions with breathing, cyanosis, and decreased urinary output; coma, accessory muscle use, cool and clammy skin, and unexplained fatigue occur as later manifestations of inadequate oxygenation.

When the patient is experiencing metabolic acidosis secondary to type 1 diabetes mellitus, what physiologic response should the nurse expect to assess in the patient? 1. Vomiting 2. Increased urination 3. Decreased heart rate 4. Rapid respiratory rate

9. Rapid respiratory rate a. When a patient with type 1 diabetes has hyperglycemia and ketonemia causing metabolic acidosis, the physiologic response is to increase the respiratory rate and tidal volume to blow off the excess CO2. Vomiting and increased urination may occur with hyperglycemia, but not as physiologic responses to metabolic acidosis. The heart rate will increase.

A 75 yar old patient who is breathing room air has the following ABG results: pH 7.40, Pa02 74mm Hg, Sa02 92%, PaCo2 40mm Hg. What is the most appropriate action by the nurse? A Document the results in the patients record B Repeat the ABGs withing an hour to validate the findings C Encourage deep breathing and coughing to open the alveoli D Initiate pulse oximetry for continuous monitoring of the patient's oxygen status

A Document the results in the patients record

After which diagnostic study should the nurse observe the patient for symptoms of a pneumothorax? A Thoracentesis B Pulmonary function test C Ventilation-perfusion scan D Positron emision tomography (PET) scan

A Thoracentesis

What should the nurse do when preparing a patient for a pulmonary angiogram? A assess the patient for iodine allergy B Implement NPO orders for 6 to 12 hours before the test. C Explain the test before the patient signs the informed consent form D Inform the patient that radiation isolation for 24 hours after the test is necessary

A assess the patient for iodine allergy

Surfactant

A lipid protein that lowers the surface tension in the alveoli. It reduces the amount of pressure needed to inflate the alveoli and makes them less likely to collapse.

Compliance

A measure of ease of expansion of the lungs.

Chemoreceptor

A receptor that responds to a change i the chemical composition of the fluid around it.

Which age related changes in the respiratory system cause decreased secretion clearance (select all that apply) A Decreased force of cough B Decreased functional cilia C Decreased chest wall compliance D Small airway closure earlier in expiration E Decreased functional immunoglobulin A

A, B

A student nurse asks the RN what cam be measured by arterial blood gas (ABG). The RN tells the student that the ABG can measure (select all that apply) A. Acid-base balance B. Oxygenation status C. Acidity of the blood D. Bicarbonate (HCO3) E. Overall balance of electrolytes in arterial blood

A. Acid-base balance B. Oxygenation status C. Acidity of the blood D. Bicarbonate (HCO3)

When teaching a patient about the most important respiratory defense mechanism distal to the respiratory bronchioles, which topic would the nurse discuss A. Alveolar macrophages B. Impaction of particles C. Reflex bronchoconstriction D. Mucociliary clearance mechanism

A. Alveolar macrophages

When assessing activity-exercise patterns related to respiratory health, the nurse inquires about A. Dyspnea during rest or exercise B. Recent weight loss or weight gain C. Ability to sleep through the entire night D. Willingness to wear 02 equipment in public

A. Dyspnea during rest or exercise

A patient with a recent history of a dry cough has had a chest x-ray that revealed the presence of nodules. In an effort to determine whether the nodules are malignant or benign, what is the primary care provider likely to order? a. Positron emission tomography (PET) b. Thoracentesis c. Pulmonary angiogram d. CT scan of the patient's chest

A. Positron emission tomography (PET) PET is used to distinguish benign and malignant pulmonary nodules. Because malignant lung cells have an increased uptake of glucose, the PET scan (which uses an IV radioactive glucose preparation) can demonstrate increased uptake of glucose in malignant lung cells. This differentiation cannot be made using CT, a pulmonary angiogram, or thoracentesis.

To promote the release of surfactant, the nurse encouraged the patient to A. Take deep breaths B. Cough five times per hour to prevent alveolar collapse C. Decrease fluid intake to reduce fluid accumulation in the alveoli D. Sit with head of bed elevated to promote air movement through the pores of Kohn

A. Take deep breaths

The nurse is preparing the patient for a diagnostic procedure to remove pleural fluid fo analysis. The nurse would prepare the patient for which test. A. Throacentesis B. Bronchoscopy C. Pulmonary angiography D. Sputum culture and sensitivity

A. Throacentesis

A patient tells the nurse that he has smoked 1 packs of cigarettes a day for 14 years. The nurse records this as _____ pack-years?

ANS: 21 1 packs of cigarettes ´ 14 years = 21 pack-years.

The nurse is percussing over the lungs of a patient with pneumonia. The nurse knows that percussion over an area of atelectasis in the lungs will reveal: a. Dullness. b. Tympany. c. Resonance. d. Hyperresonance.

ANS: A A dull percussion note signals an abnormal density in the lungs, as with pneumonia, pleural effusion, atelectasis, or a tumor.

A 70-year-old patient is being seen in the clinic for severe exacerbation of his heart failure. Which of these findings is the nurse most likely to observe in this patient? a. Shortness of breath, orthopnea, paroxysmal nocturnal dyspnea, and ankle edema b. Rasping cough, thick mucoid sputum, wheezing, and bronchitis c. Productive cough, dyspnea, weight loss, anorexia, and tuberculosis d. Fever, dry nonproductive cough, and diminished breath sounds

ANS: A A person with heart failure often exhibits increased respiratory rate, shortness of breath on exertion, orthopnea, paroxysmal nocturnal dyspnea, nocturia, ankle edema, and pallor in light-skinned individuals. A patient with rasping cough, thick mucoid sputum, and wheezing may have bronchitis. Productive cough, dyspnea, weight loss, and dyspnea indicate tuberculosis; fever, dry nonproductive cough, and diminished breath sounds may indicate Pneumocystis jiroveci (P. carinii) pneumonia (see Table 18-8).

An adult patient with a history of allergies comes to the clinic complaining of wheezing and difficulty in breathing when working in his yard. The assessment findings include tachypnea, the use of accessory neck muscles, prolonged expiration, intercostal retractions, decreased breath sounds, and expiratory wheezes. The nurse interprets that these assessment findings are consistent with: a. Asthma. b. Atelectasis. c. Lobar pneumonia. d. Heart failure.

ANS: A Asthma is allergic hypersensitivity to certain inhaled particles that produces inflammation and a reaction of bronchospasm, which increases airway resistance, especially during expiration. An increased respiratory rate, the use of accessory muscles, a retraction of the intercostal muscles, prolonged expiration, decreased breath sounds, and expiratory wheezing are all characteristics of asthma.

During auscultation of breath sounds, the nurse should correctly use the stethoscope in which of the following ways? a. Listening to at least one full respiration in each location b. Listening as the patient inhales and then going to the next site during exhalation c. instructing the patient to breathe in and out rapidly while listening to the breath sounds d. If the patient is modest, listening to sounds over his or her clothing or hospital gown

ANS: A During auscultation of breath sounds with a stethoscope, listening to one full respiration in each location is important. During the examination, the nurse should monitor the breathing and offer times for the person to breathe normally to prevent possible dizziness.

A nurse notices a patient's chest wall moving in during inspiration and out during expiration. What additional assessment must the nurse perform immediately? a. Palpate for tracheal deviation. b. Auscultate for bronchovesicular breath sounds in the lung periphery. c. Palpate posterior thoracic muscles for tenderness. d. Auscultate for absence of breath sounds in the lung periphery.

ANS: A Feedback A Chest wall moving in during inspiration and out during expiration is paradoxical chest wall movement. It can be caused by a tension pneumothorax, which increases intrathoracic pressure in the thorax, causing tracheal deviation and indicating mediastinal shift. B Tension pneumothorax does not create bronchovesicular breath sounds in the lung periphery. C This is performed when the patient has air in the subcutaneous tissue or pleural friction rub. D Absent breath sounds may be found in pneumothorax, but if the patient has a tension pneumothorax, tracheal deviation is a more important sign.

During the problem-based history, a patient reports coughing up sputum when lying on the right side, but not when lying on the back or left side. The nurse suspects this patient may have a lung abscess. What additional question does the nurse ask to gather more data? a. "Does the sputum have an odor?" b. "Do you have chest pain when you take a deep breath?" c. "Have you also experienced tightness in your chest?" d. "Have you coughed up any blood?"

ANS: A Feedback A Sputum with odor and sputum production with change of position is associated with lung abscess or bronchiectasis. B Chest pain on deep breathing is associated with pleural lining irritation. C Tightness in the chest is associated with asthma. D Coughing up rust-colored sputum is associated with pneumonia, but coughing up blood may be associated with lung cancer.

After taking a brief health history, a nurse needs to complete a focused assessment on which patient? a. A male who works as a painter b. A male who plays basketball and hockey c. A female who recently moved into a college dormitory d. A female who has a history of gout

ANS: A Feedback A The fumes and chemicals from the paint may expose the patient to respiratory irritants. A baseline pulmonary assessment needs to be documented. B This patient is not at risk for pulmonary disease. C This patient is not at risk for pulmonary disease. D This patient is not at risk for pulmonary disease.

Which patient should the nurse assess first? a. The patient whose respiratory rate is 26 breaths per minute and whose trachea deviates to the right. b. The patient who has pleuritic chest pain, bilateral crackles, a productive cough of yellow sputum, and fever. c. The patient who is short of breath, using pursed-lip breathing, and in a tripod position. d. The patient whose respiratory rate is 20 breaths/min, and has 8-word dyspnea and expiratory wheezes.

ANS: A Feedback A This is a description of a left tension pneumothorax. The key manifestation is deviation of the trachea from midline, which indicates high intrathoracic pressure from the left that is pushing the mediastinum out of alignment. The respiratory rate indicates tachypnea. B This is a description of a patient with pneumonia who needs to be examined, but this is not a life-threatening condition. C This is a description of a patient with emphysema, a chronic disease. This patient may have these manifestations frequently and does not need to be examined immediately. D This is a description of a patient who is having an asthma attack, but it is not a life threatening attack; the respiratory rate is the upper limits of normal; the dyspnea is abnormal, but not far from normal; and the wheezing is on expiration only.

A patient has right lower lobe pneumonia, creating a consolidation in that lung. In assessing for vocal fremitus, the nurse found increased fremitus over the right lower lung. What finding does the nurse anticipate when assessing vocal resonance to confirm the consolidation? a. Bronchophony reveals the patient's spoken "99" as clear and loud. b. No sounds are expected since sounds cannot be transmitted through consolidation. c. Egophony reveals indistinguishable sounds when the patient says "e-e-e." d. Whispered pectoriloquy reveals a muffled sound when the patient says "1-2-3."

ANS: A Feedback A This is an abnormal finding and occurs in consolidation. B The abnormal finding is hearing a clear sound. C This is a normal finding. D This is a normal finding.

A nurse had previously heard crackles over both lungs of a patient. As the patient improves, what lung sounds does the nurse expect to hear in the patient's lungs? a. Vesicular breath sounds heard in peripheral lung fields b. Bronchial breath sounds heard over the bronchi c. Bronchovesicular breath sounds heard over the apices d. Rhonchi heard over the main bronchi

ANS: A Feedback A Vesicular breath sounds heard in peripheral lung fields are an expected finding for healthy lungs. B Bronchial breath sounds are heard over the trachea. C Bronchovesicular breath sounds are heard anteriorly near the sternal border first and second intercostals space. D Rhonchi are adventitious sounds indicating secretions in the bronchi.

When assessing tactile fremitus, the nurse recalls that it is normal to feel tactile fremitus most intensely over which location? a. Between the scapulae b Third intercostal space, MCL c. Fifth intercostal space, midaxillary line (MAL) d. Over the lower lobes, posterior side

ANS: A Normally, fremitus is most prominent between the scapulae and around the sternum. These sites are where the major bronchi are closest to the chest wall. Fremitus normally decreases as one progresses down the chest because more tissue impedes sound transmission.

. A nurse examines a patient with a pleural effusion and finds decreased fremitus. What additional abnormal finding should the nurse anticipate during further examination? a. An increase in the anteroposterior to lateral ratio b. Hyperresonance over the affected area c. Absent breath sounds in the affected area d. Increased vocal fremitus over the affected area

ANS: C Feedback A An increase in the anteroposterior to lateral ratio occurs in overinflated lungs as in emphysema. B Hyperresonance over the affected area occurs in overinflated lungs as in emphysema. C Absent breath sound in the affected area is anticipated because the fluid in the pleural space prevents breath sounds from being heard. D Increased vocal fremitus over the affected area is associated with consolidation that occurs with pneumonia or tumor. Fremitus is decreased to absent in pleural effusion.

The nurse is auscultating the lungs of a patient who had been sleeping and notices short, popping, crackling sounds that stop after a few breaths. The nurse recognizes that these breath sounds are: a. Atelectatic crackles that do not have a pathologic cause. b. Fine crackles and may be a sign of pneumonia. c. Vesicular breath sounds. d. Fine wheezes

ANS: A One type of adventitious sound, atelectatic crackles, does not have a pathologic cause. They are short, popping, crackling sounds that sound similar to fine crackles but do not last beyond a few breaths. When sections of alveoli are not fully aerated (as in people who are asleep or in older adults), they deflate slightly and accumulate secretions. Crackles are heard when these sections are expanded by a few deep breaths. Atelectatic crackles are heard only in the periphery, usually in dependent portions of the lungs, and disappear after the first few breaths or after a cough.

The nurse is observing the auscultation technique of another nurse. The correct method to use when progressing from one auscultatory site on the thorax to another is _______ comparison. a. Side-to-side b. Top-to-bottom c. Posterior-to-anterior d. Interspace-by-interspace

ANS: A Side-to-side comparison is most important when auscultating the chest. The nurse should listen to at least one full respiration in each location. The other techniques are not correct.

The primary muscles of respiration include the: a. Diaphragm and intercostals. b. Sternomastoids and scaleni. c. Trapezii and rectus abdominis. d. External obliques and pectoralis major.

ANS: A The major muscle of respiration is the diaphragm. The intercostal muscles lift the sternum and elevate the ribs during inspiration, increasing the anteroposterior diameter. Expiration is primarily passive. Forced inspiration involves the use of other muscles, such as the accessory neck muscles—sternomastoid, scaleni, and trapezii muscles. Forced expiration involves the abdominal muscles.

Which of these statements is true regarding the vertebra prominens? The vertebra prominens is: a.The spinous process of C7. b.Usually nonpalpable in most individuals. c.Opposite the interior border of the scapula. d.Located next to the manubrium of the sternum.

ANS: A The spinous process of C7 is the vertebra prominens and is the most prominent bony spur protruding at the base of the neck. Counting ribs and intercostal spaces on the posterior thorax is difficult because of the muscles and soft tissue. The vertebra prominens is easier to identify and is used as a starting point in counting thoracic processes and identifying landmarks on the posterior chest.

The nurse is listening to the breath sounds of a patient with severe asthma. Air passing through narrowed bronchioles would produce which of these adventitious sounds? a. Wheezes b. Bronchial sounds c. Bronchophony d. Whispered pectoriloquy

ANS: A Wheezes are caused by air squeezed or compressed through passageways narrowed almost to closure by collapsing, swelling, secretions, or tumors, such as with acute asthma or chronic emphysema.

During auscultation of the lungs, the nurse expects decreased breath sounds to be heard in which situation? a. When the bronchial tree is obstructed b. When adventitious sounds are present c. In conjunction with whispered pectoriloquy d. In conditions of consolidation, such as pneumonia

ANS: A Decreased or absent breath sounds occur when the bronchial tree is obstructed, as in emphysema, and when sound transmission is obstructed, as in pleurisy, pneumothorax, or pleural effusion.

On examination, a nurse finds the patient has a productive cough with green sputum and inspiratory crackles. What other findings does this nurse expect during the examination? Select all that apply. a. Dull tones to percussion b. Increased vibration on vocal fremitus c. Fever d. Decreased diaphragmatic excursion e. A sharp, abrupt pain reported when patient breathes deeply f. Muffled sounds heard when the patient says "e-e-e"

ANS: A, B, C, E Correct: These abnormal findings are consistent with consolidation that may occur with pneumonia. Incorrect: Decreased diaphragmatic excursion occurs when the lung is overinflated as in emphysema. Muffled sounds when the patient says "e-e-e" is an expected finding. With a consolidation, the sound of "e-e-e" would be clear.

The nurse is assessing voice sounds during a respiratory assessment. Which of these findings indicates a normal assessment? Select all that apply. a. Voice sounds are faint, muffled, and almost inaudible when the patient whispers "one, two, three" in a very soft voice. b. As the patient repeatedly says "ninety-nine," the examiner clearly hears the words "ninety-nine." c. When the patient speaks in a normal voice, the examiner can hear a sound but cannot exactly distinguish what is being said. d. As the patient says a long "ee-ee-ee" sound, the examiner also hears a long "ee-ee-ee" sound. e. As the patient says a long "ee-ee-ee" sound, the examiner hears a long "aaaaaa" sound.

ANS: A, C, D As a patient repeatedly says "ninety-nine," normally the examiner hears voice sounds but cannot distinguish what is being said. If a clear "ninety-nine" is auscultated, then it could indicate increased lung density, which enhances the transmission of voice sounds, which is a measure of bronchophony. When a patient says a long "ee-ee-ee" sound, normally the examiner also hears a long "ee-ee-ee" sound through auscultation, which is a measure of egophony. If the examiner hears a long "aaaaaa" sound instead, this sound could indicate areas of consolidation or compression. With whispered pectoriloquy, as when a patient whispers a phrase such as "one-two-three," the normal response when auscultating voice sounds is to hear sounds that are faint, muffled, and almost inaudible. If the examiner clearly hears the whispered voice, as if the patient is speaking through the stethoscope, then consolidation of the lung fields may exist.

What are the functions of the upper airways? Select all that apply. a. Conduct air to lower airway. b. Provide area for gas exchange. c. Prevent foreign matter from entering respiratory system. d. Warm, humidify, and filter air entering lungs. e. Provide transportation of oxygen and carbon dioxide between alveoli and cells.

ANS: A, C, D Correct: These are functions of the upper airway. Incorrect: Gas exchange occurs in the alveoli. The cardiovascular system provides transportation of oxygen and carbon dioxide between alveoli and cells.

A patient has been admitted to the emergency department for a suspected drug overdose. His respirations are shallow, with an irregular pattern, with a rate of 12 respirations per minute. The nurse interprets this respiration pattern as which of the following? a. Bradypnea b. Cheyne-Stokes respirations c. Hypoventilation d. Chronic obstructive breathing

ANS: C Hypoventilation is characterized by an irregular, shallow pattern, and can be caused by an overdose of narcotics or anesthetics. Bradypnea is slow breathing, with a rate less than 10 respirations per minute. (See Table 18-4 for descriptions of Cheyne-Stokes respirations and chronic obstructive breathing.)

4. A nurse is assessing the respiratory system of a healthy adult. Which findings does this nurse expect to find? Select all that apply. a. Thoracic expansion that is symmetric bilaterally b. Respiratory rate of 24 breaths/min c. Bronchophony revealing clear voice sounds d. Breath sounds clear with vesicular breath sounds heard over most lung fields e. Anteroposterior diameter of the chest about a 1:2 ratio of anteroposterior to lateral diameter f. Symmetric thorax with ribs sloping downward at about 45 degrees relative to the spine

ANS: A, D, E, F Correct: These are expected findings from a lung and respiratory assessment of a healthy adult. Incorrect: A respiratory rate of 24 breaths/min is considered tachypnea. Bronchophony revealing clear voice sounds is not performed unless there is an indication of consolidation of the lung, or if there was an abnormal finding of tactile fremitus. The expected finding is muffled voiced sounds rather than clear.

A patient comes to the clinic complaining of a cough that is worse at night but not as bad during the day. The nurse recognizes that this cough may indicate: a. Pneumonia. b. Postnasal drip or sinusitis. c. Exposure to irritants at work. d. Chronic bronchial irritation from smoking.

ANS: B A cough that primarily occurs at night may indicate postnasal drip or sinusitis. Exposure to irritants at work causes an afternoon or evening cough. Smokers experience early morning coughing. Coughing associated with acute illnesses such as pneumonia is continuous throughout the day.

A patient with pleuritis has been admitted to the hospital and complains of pain with breathing. What other key assessment finding would the nurse expect to find upon auscultation? a. Stridor b. Friction rub c. Crackles d. Wheezing

ANS: B A patient with pleuritis will exhibit a pleural friction rub upon auscultation. This sound is made when the pleurae become inflamed and rub together during respiration. The sound is superficial, coarse, and low-pitched, as if two pieces of leather are being rubbed together. Stridor is associated with croup, acute epiglottitis in children, and foreign body inhalation. Crackles are associated with pneumonia, heart failure, chronic bronchitis, and other diseases (see Table 18-6). Wheezes are associated with diffuse airway obstruction caused by acute asthma or chronic emphysema.

During palpation of the anterior chest wall, the nurse notices a coarse, crackling sensation over the skin surface. On the basis of these findings, the nurse suspects: a. Tactile fremitus. b. Crepitus. c. Friction rub. d. Adventitious sounds.

ANS: B Crepitus is a coarse, crackling sensation palpable over the skin surface. It occurs in subcutaneous emphysema when air escapes from the lung and enters the subcutaneous tissue, such as after open thoracic injury or surgery.

A nurse auscultates low-pitched, coarse snoring sounds in a patient's lungs during inhalation. What is the most appropriate action for the nurse to take at this time? a. Palpate the posterior thorax for vocal fremitus. b. Ask the patient to cough and repeat auscultation. c. Auscultate the posterior thorax for vocal sounds. d. Percuss the posterior thorax for tone.

ANS: B Feedback A An abnormal vocal fremitus (decreased or increased vibrations) is not expected for this patient. B The sounds indicate rhonchi, or secretions in the bronchi. The first action to take is to determine if the rhonchi clear with coughing. If the rhonchi clear, there is no need to further investigate this finding. C Abnormal vocal sounds (clear and loud sounds) are not expected for this patient.

During an assessment, the nurse knows that expected assessment findings in the normal adult lung include the presence of: a.Adventitious sounds and limited chest expansion bIncreased tactile fremitus and dull percussion tones. c.Muffled voice sounds and symmetric tactile fremitus. d.Absent voice sounds and hyperresonant percussion tones.

ANS: C Normal lung findings include symmetric chest expansion, resonant percussion tones, vesicular breath sounds over the peripheral lung fields, muffled voice sounds, and no adventitious sounds.

The nurse is comparing pitch and duration of the various types of a patient's breath sounds and recognizes which one of these as an expected finding? a. Bronchial sounds are low-pitched and have a 2:1 inspiratory-versus-expiratory ratio. b. Bronchovesicular sounds have a moderate pitch and 1:1 expiratory-versus-inspiratory ratio. c. Vesicular breath sounds are high-pitched and have a 1:2 inspiratory-versus-expiratory ratio. d. Wheezes are low-pitched and have a 2.5:1 inspiratory-versus-expiratory ratio.

ANS: B Feedback A Bronchial sounds are high pitched with a duration of 1:2 inspiration-to-expiration is the correct statement. B Bronchovesicular sounds having a moderate pitch and 1:1 expiratory-versus-inspiratory ratio is a normal finding. C Vesicular sounds are low pitched with a duration of 2.5:1 inspiration-to-expiration is the correct statement. D Wheezes are high-pitched and have no specific duration because they are adventitious sounds.

A patient is suspected of having a lung consolidation. A nurse uses the three techniques for assessing vocal resonance in this patient. What is the expected finding among the three procedures that will help eliminate consolidation as a problem? a. The nurse documents clearly hearing the patient say "99." b. The nurse documents hearing muffled sounds when the patient says "1-2-3." c. The nurse documents hearing no sounds when the patient says "e-e-e." d. The nurse documents clearly hearing the patient say "a-a-a."

ANS: B Feedback A Clear sounds are heard when a consolidation is present. B Muffled sounds of "1-2-3," "e-e-e," or "99" are heard when no consolidation is found. C Clear sounds are heard when a consolidation is present. D Clear sounds are heard when a consolidation is present.

A patient reports a productive cough with yellow sputum, fever, and a sharp pain when taking a deep breath to cough. Based on these data, what abnormal finding will the nurse anticipate on examination? a. Decreased breath sounds on auscultation b. Increased tactile fremitus and dull percussion tones c. Inspiratory wheezing found on auscultation d. Muffled sounds heard when the patient says "e-e-e"

ANS: B Feedback A Decreased breath sounds on auscultation is consistent with emphysema or atelectasis when alveoli are narrowed or destroyed. B The data describe purulent sputum and inflammation of the pleura that may occur in pneumonia. Additional findings include increased tactile fremitus and dull percussion tones, indicating congested or consolidated lung tissues. C Inspiratory wheezing found on auscultation is consistent with narrowing of bronchi that may occur in asthma. D Muffled sounds heard when the patient says "e-e-e" is a normal finding on vocal resonance (bronchophony or egophony).

A nurse is assessing a patient who was diagnosed with emphysema and chronic bronchitis 5 years ago. During the assessment of this patient's integumentary system, what finding should the nurse correlate to this respiratory disease? a. Dry, flaky skin b. Clubbing of the fingers c. Hypertrophy of the nails d. Hair loss from the scalp

ANS: B Feedback A Dry, flaky skin occurs with dehydration. B Clubbing of the fingers develops due to chronic hypoxemia, which occurs in chronic obstructive pulmonary disease. C Hypertrophy of the nails occurs with repeated trauma. D Hair loss from the scalp is alopecia, which occurs with many systemic diseases, but not chronic pulmonary disease.

A nurse suspects a patient has a chest wall injury and wants to collect more data about thoracic expansion. Which is the appropriate technique to use? a. Place the palmar side of each hand against the lateral thorax at the level of the waist, ask the patient to take a deep breath, and observe lateral movement of the hands. b. Place both thumbs on either side of the patient's T9 to T10 spinal processes, extend fingers laterally, ask the patient to take a deep breath, and observe lateral movement of the thumbs. c. Place both thumbs on either side of the patient's T7 to T8 spinal processes, extend fingers laterally, ask the patient to exhale deeply, and observe lateral inward movement of the thumbs. d. Place the palmar side of each hand on the shoulders of the patient, ask the patient to sit up straight and take a deep breath, and observe symmetric movement of the shoulders.

ANS: B Feedback A The palms of the hands are not used and hands are not placed on the lateral thorax. B This is the correct technique to assess thoracic expansion. C The thoracic level is too high and the patient does not exhale. D The hands are not placed on the shoulders.

Which question will give the nurse additional information about the nature of a patient's dyspnea? a. "How often do you see the physician?" b. "How has this condition affected your day-to-day activities?" c. "Do you have a cough that occurs with the dyspnea?" d. "Does your heart rate increase when you are short of breath?"

ANS: B Feedback A This question does not relate specifically to the patient's dyspnea. B This question provides data about the severity of the dyspnea and what actions the patient has taken to cope with the dyspnea on a daily basis. C This question provides data, but does not give additional facts about the patient's dyspnea. D This is a closed-ended question that does not collect additional data about this episode of dyspnea.

A nurse is auscultating the lungs of a healthy male patient and hears crackles on inspiration. What action can the nurse take to ensure this is an accurate finding? a. Make sure the bell of the stethoscope is used, rather than the diaphragm. b. Hold stethoscope firmly to prevent movement when placed over chest hair. c. Ask the patient not to talk while the nurse is listening to the lungs. d. Change the patient's position to ensure accurate sounds.

ANS: B Feedback A Using the bell will provide inaccurate sounds, but not mimic crackles. B The stethoscope moving even slightly on chest hair can mimic the sound of crackles. C When the patient talks during auscultation, it does interfere with data collection, but the sound is a muffled voice. D Changing the position will not affect the outcome of the assessment if the initial problem remains.

Which statement about the apices of the lungs is true? The apices of the lungs: a.Are at the level of the second rib anteriorly. b.Extend 3 to 4 cm above the inner third of the clavicles. c.are located at the sixth rib anteriorly and the eighth rib laterally. d. Rest on the diaphragm at the fifth intercostal space in the midclavicular line (MCL).

ANS: B The apex of the lung on the anterior chest is 3 to 4 cm above the inner third of the clavicles. On the posterior chest, the apices are at the level of C7.

The nurse knows that a normal finding when assessing the respiratory system of an older adult is: a. Increased thoracic expansion. b. Decreased mobility of the thorax. c. Decreased anteroposterior diameter. d. Bronchovesicular breath sounds throughout the lungs.

ANS: B The costal cartilages become calcified with aging, resulting in a less mobile thorax. Chest expansion may be somewhat decreased, and the chest cage commonly shows an increased anteroposterior diameter.

. A mother brings her 3-month-old infant to the clinic for evaluation of a cold. She tells the nurse that he has had "a runny nose for a week." When performing the physical assessment, the nurse notes that the child has nasal flaring and sternal and intercostal retractions. The nurse's next action should be to: a. Assure the mother that these signs are normal symptoms of a cold. b. Recognize that these are serious signs, and contact the physician. c. Ask the mother if the infant has had trouble with feedings. d. Perform a complete cardiac assessment because these signs are probably indicative of early heart failure.

ANS: B The infant is an obligatory nose breather until the age of 3 months. Normally, no flaring of the nostrils and no sternal or intercostal retraction occurs. Significant retractions of the sternum and intercostal muscles and nasal flaring indicate increased inspiratory effort, as in pneumonia, acute airway obstruction, asthma, and atelectasis; therefore, immediate referral to the physician is warranted. These signs do not indicate heart failure, and an assessment of the infant's feeding is not a priority at this time.

During an examination of the anterior thorax, the nurse is aware that the trachea bifurcates anteriorly at the: a.Costal angle. b.Sternal angle. c.Xiphoid process. d.Suprasternal notch.

ANS: B The sternal angle marks the site of tracheal bifurcation into the right and left main bronchi; it corresponds with the upper borders of the atria of the heart, and it lies above the fourth thoracic vertebra on the back.

During an assessment of an adult, the nurse has noted unequal chest expansion and recognizes that this occurs in which situation? a. In an obese patient b. When part of the lung is obstructed or collapsed c. When bulging of the intercostal spaces is present d. When accessory muscles are used to augment respiratory effort

ANS: B Unequal chest expansion occurs when part of the lung is obstructed or collapsed, as with pneumonia, or when guarding to avoid postoperative incisional pain.

A teenage patient comes to the emergency department with complaints of an inability to breathe and a sharp pain in the left side of his chest. The assessment findings include cyanosis, tachypnea, tracheal deviation to the right, decreased tactile fremitus on the left, hyperresonance on the left, and decreased breath sounds on the left. The nurse interprets that these assessment findings are consistent with: a. Bronchitis. b. Pneumothorax. c. Acute pneumonia. d. Asthmatic attack.

ANS: B With a pneumothorax, free air in the pleural space causes partial or complete lung collapse. If the pneumothorax is large, then tachypnea and cyanosis are evident. Unequal chest expansion, decreased or absent tactile fremitus, tracheal deviation to the unaffected side, decreased chest expansion, hyperresonant percussion tones, and decreased or absent breath sounds are found with the presence of pneumothorax.

On inspection, the nurse finds the patient's anteroposterior diameter of the chest to be the same as the lateral diameter. What other findings does this nurse expect during the examination? Select all that apply. a. Inspiratory wheezing found on auscultation b. Hyperresonance heard on percussion c. Decreased breath sounds heard on auscultation d. Deceased diaphragmatic excursion on percussion e. A sharp, abrupt pain reported when the patient breathes deeply f. Decreased to absent vibration on vocal fremitus

ANS: B, C, D, F Correct: These are all indications of enlargement or destruction of alveoli that occurs in emphysema. Air is trapped, which increases the anteroposterior to lateral diameter creating a barrel chest, and pushes the diaphragm down decreasing the excursion and causing hyperresonance. The destroyed alveoli decrease the breath sounds and create absent vibration on vocal fremitus. Incorrect: Inspiratory wheezing found on auscultation indicates narrowed airways as found in asthma. A sharp, abrupt pain reported when the patient breathes deeply is pleuritic chest pain associated with pleural lining irritation that may occur in a patient with pleurisy or pneumonia

The nurse is reviewing the characteristics of breath sounds. Which statement about bronchovesicular breath sounds is true? Bronchovesicular breath sounds are: a. Musical in quality. b. Usually caused by a pathologic disease. c. Expected near the major airways. d. Similar to bronchial sounds except shorter in duration.

ANS: C Bronchovesicular breath sounds are heard over major bronchi where fewer alveoli are located posteriorly—between the scapulae, especially on the right; and anteriorly, around the upper sternum in the first and second intercostal spaces. The other responses are not correct.

When assessing the respiratory system of a 4-year-old child, which of these findings would the nurse expect? a. Crepitus palpated at the costochondral junctions b. No diaphragmatic excursion as a result of a child's decreased inspiratory volume c. Presence of bronchovesicular breath sounds in the peripheral lung fields d. Irregular respiratory pattern and a respiratory rate of 40 breaths per minute at rest

ANS: C Bronchovesicular breath sounds in the peripheral lung fields of the infant and young child up to age 5 or 6 years are normal findings. Their thin chest walls with underdeveloped musculature do not dampen the sound, as do the thicker chest walls of adults; therefore, breath sounds are loud and harsh.

A woman in her 26th week of pregnancy states that she is "not really short of breath" but feels that she is aware of her breathing and the need to breathe. What is the nurse's best reply? a. "The diaphragm becomes fixed during pregnancy, making it difficult to take in a deep breath." b. "The increase in estrogen levels during pregnancy often causes a decrease in the diameter of the rib cage and makes it difficult to breathe." c. "What you are experiencing is normal. Some women may interpret this as shortness of breath, but it is a normal finding and nothing is wrong." d. "This increased awareness of the need to breathe is normal as the fetus grows because of the increased oxygen demand on the mother's body, which results in an increased respiratory rate."

ANS: C During pregnancy, the woman may develop an increased awareness of the need to breathe. Some women may interpret this as dyspnea, although structurally nothing is wrong. Increases in estrogen relax the chest cage ligaments, causing an increase in the transverse diameter. Although the growing fetus increases the oxygen demand on the mother's body, this increased demand is easily met by the increasing tidal volume (deeper breathing). Little change occurs in the respiratory rate.

A nurse palpating the chest of a patient finds increased fremitus bilaterally. What is the significance of this finding? a. An expected finding b. Chronic obstructive pulmonary disease c. Bilateral pneumonia d. Bilateral pneumothorax

ANS: C Feedback A An increase in fremitus from normal is not an expected finding. B Air trapping in chronic obstructive pulmonary disease causes a decreased fremitus. C Increased fremitus occurs when lung tissues are congested or consolidated, which may occur in patients who have pneumonia or a tumor. D Air in the pleural space causes a decreased fremitus.

On inspection, a nurse finds the patient's anteroposterior diameter of the chest to be the same as the lateral diameter. Based on this finding, what additional data does the nurse anticipate? a. Increased vocal fremitus on palpation b. Dull tones heard on percussion c. Decreased breath sounds on auscultation d. Complaint of sharp chest pain on inspiration

ANS: C Feedback A Increased fremitus occurs when the vibrations feel enhanced. This is found when lung tissues are congested or consolidated, which may occur in patients who have pneumonia or a tumor. B Dull tones may be heard in patients with pneumonia, pleural effusion, or atelectasis. C The equal anteroposterior and lateral diameters of the chest indicate air trapping from enlarged or destroyed alveoli. This air trapping causes decreased to absent breath sounds on auscultation. D Complaint of sharp chest pain on inspiration is pleuritic chest pain associated with pleural lining irritation and may occur in a patient with pleurisy or pneumonia.

A nurse inspects a patient's hands and notices clubbing of the fingers. The nurse correlates this finding with what condition? a. Pulmonary infection b. Trauma to the thorax c. Chronic hypoxemia d. Allergic reaction

ANS: C Feedback A Pulmonary infection is acute and not associated with chronic hypoxia. B Trauma to the thorax is acute and not associated with chronic hypoxia. C Clubbing develops due to chronic hypoxemia, which occurs in chronic obstructive pulmonary disease. D Allergic reaction is acute and not associated with chronic hypoxia.

A patient tells the nurse that she has smoked two packs of cigarettes a day for 20 years. The nurse records this as how many pack-years? a. 10 b. 20 c. 40 d. 60

ANS: C Feedback A This incorrect calculation was made by dividing 20 years by 2 packs. B This is correct if the patient smoked 1 pack per day for 20 years. C Two packs of cigarettes ´ 20 years = 40 pack-years. D This is correct if the patient smoked 3 packs per day for 20 years or 2 packs a day for 30 years.

A patient complains of shortness of breath and having to sleep on three pillows to breathe comfortably at night. During the nurse's examination, what findings will suggest that the cause of this patient's dyspnea is due to heart disease rather than respiratory disease? a. Increased anteroposterior diameter b. Clubbing of the fingers c. Bilateral peripheral edema d. Increased tactile fremitus

ANS: C Feedback A This is seen with lung hyperinflation and may be associated with emphysema. B This is associated with chronic hypoxia and may be associated with cystic fibrosis or chronic obstructive pulmonary disease. C This indicates heart failure; dyspnea occurs because the heart cannot adequately perfuse the lungs. D This occurs when vibrations are enhanced and is associated with consolidation that may occur in pneumonia or tumor.

A nurse is assessing for vocal (tactile) fremitus on a patient with pulmonary edema. Which is the appropriate technique to use? a. Systematically percuss the posterior chest wall following the same pattern that is used for auscultation and listen for a change in tone from resonant to dull. b. Place the pads of the fingers on the right and left thoraces and palpate the texture and consistency of the skin feeling for a crackly sensation under the fingers. c. Place the palms of the hands on the right and left thoraces, ask the patient to say "99," and feel for vibrations. d. Place both thumbs on either side of the patient's spinal processes, extend fingers laterally, ask the patient to take a deep breath, and feel for vibrations.

ANS: C Feedback A This is the technique for percussing the thorax for tones. B This is the technique for detecting crepitus. C This is the correct technique for vocal fremitus. D This is not the correct technique.

A patient has been admitted to the emergency department with a possible medical diagnosis of pulmonary embolism. The nurse expects to see which assessment findings related to this condition? a. Absent or decreased breath sounds b. Productive cough with thin, frothy sputum c. Chest pain that is worse on deep inspiration and dyspnea d. diffuse infiltrates with areas of dullness upon percussion

ANS: C Findings for pulmonary embolism include chest pain that is worse on deep inspiration, dyspnea, apprehension, anxiety, restlessness, partial arterial pressure of oxygen (PaO2) less than 80 mm Hg, diaphoresis, hypotension, crackles, and wheezes.

The nurse knows that auscultation of fine crackles would most likely be noticed in: a. A healthy 5-year-old child. b. A pregnant woman. c. The immediate newborn period. d. Association with a pneumothorax.

ANS: C Fine crackles are commonly heard in the immediate newborn period as a result of the opening of the airways and a clearing of fluid. Persistent fine crackles would be noticed with pneumonia, bronchiolitis, or atelectasis.

The nurse is auscultating the chest in an adult. Which technique is correct? a. Instructing the patient to take deep, rapid breaths b. Instructing the patient to breathe in and out through his or her nose c. Firmly holding the diaphragm of the stethoscope against the chest d. Lightly holding the bell of the stethoscope against the chest to avoid friction

ANS: C Firmly holding the diaphragm of the stethoscope against the chest is the correct way to auscultate breath sounds. The patient should be instructed to breathe through his or her mouth, a little deeper than usual, but not to hyperventilate.

The nurse is reviewing the technique of palpating for tactile fremitus with a new graduate. Which statement by the graduate nurse reflects a correct understanding of tactile fremitus? "Tactile fremitus: a. "Is caused by moisture in the alveoli." b. "Indicates that air is present in the subcutaneous tissues." c. "Is caused by sounds generated from the larynx." d. "Reflects the blood flow through the pulmonary arteries."

ANS: C Fremitus is a palpable vibration. Sounds generated from the larynx are transmitted through patent bronchi and the lung parenchyma to the chest wall where they are felt as vibrations. Crepitus is the term for air in the subcutaneous tissues.

The nurse is reviewing the technique of palpating for tactile fremitus with a new graduate. Which statement by the graduate nurse reflects a correct understanding of tactile fremitus? "Tactile fremitus: a. "Is caused by moisture in the alveoli." b. "Indicates that air is present in the subcutaneous tissues." c. "Is caused by sounds generated from the larynx." d. "Reflects the blood flow through the pulmonary arteries."

ANS: C Fremitus is a palpable vibration. Sounds generated from the larynx are transmitted through patent bronchi and the lung parenchyma to the chest wall where they are felt as vibrations. Crepitus is the term for air in the subcutaneous tissues.

Hyperresonance

air trapping

During auscultation of the lungs of an adult patient, the nurse notices the presence of bronchophony. The nurse should assess for signs of which condition? a. Airway obstruction b. Emphysema c. Pulmonary consolidation d. Asthma

ANS: C Pathologic conditions that increase lung density, such as pulmonary consolidation, will enhance the transmission of voice sounds, such as bronchophony (see Table 18-7).

. A 35-year-old recent immigrant is being seen in the clinic for complaints of a cough that is associated with rust-colored sputum, low-grade afternoon fevers, and night sweats for the past 2 months. The nurse's preliminary analysis, based on this history, is that this patient may be suffering from: a. Bronchitis. b. Pneumonia. c. Tuberculosis. d. Pulmonary edema.

ANS: C Sputum is not diagnostic alone, but some conditions have characteristic sputum production. Tuberculosis often produces rust-colored sputum in addition to other symptoms of night sweats and low-grade afternoon fevers (see Table 18-8).

A 65-year-old patient with a history of heart failure comes to the clinic with complaints of "being awakened from sleep with shortness of breath." Which action by the nurse is most appropriate? a. Obtaining a detailed health history of the patient's allergies and a history of asthma b. Telling the patient to sleep on his or her right side to facilitate ease of respirations c. Assessing for other signs and symptoms of paroxysmal nocturnal dyspnea d. Assuring the patient that paroxysmal nocturnal dyspnea is normal and will probably resolve within the next week

ANS: C The patient is experiencing paroxysmal nocturnal dyspnea—being awakened from sleep with shortness of breath and the need to be upright to achieve comfort.

When performing a respiratory assessment on a patient, the nurse notices a costal angle of approximately 90 degrees. This characteristic is: a.Observed in patients with kyphosis. b.Indicative of pectus excavatum. c.A normal finding in a healthy adult. d.An expected finding in a patient with a barrel chest.

ANS: C The right and left costal margins form an angle where they meet at the xiphoid process. Usually, this angle is 90 degrees or less. The angle increases when the rib cage is chronically overinflated, as in emphysema.

When auscultating the lungs of an adult patient, the nurse notes that low-pitched, soft breath sounds are heard over the posterior lower lobes, with inspiration being longer than expiration. The nurse interprets that these sounds are: a. Normally auscultated over the trachea. b. Bronchial breath sounds and normal in that location. c. Vesicular breath sounds and normal in that location. d. Bronchovesicular breath sounds and normal in that location.

ANS: C Vesicular breath sounds are low-pitched, soft sounds with inspiration being longer than expiration. These breath sounds are expected over the peripheral lung fields where air flows through smaller bronchioles and alveoli.

During percussion, the nurse knows that a dull percussion note elicited over a lung lobe most likely results from: a. Shallow breathing. b. Normal lung tissue. c. Decreased adipose tissue. d. Increased density of lung tissue.

ANS: D A dull percussion note indicates an abnormal density in the lungs, as with pneumonia, pleural effusion, atelectasis, or a tumor. Resonance is the expected finding in normal lung tissue.

A patient has a long history of chronic obstructive pulmonary disease (COPD). During the assessment, the nurse will most likely observe which of these? a. Unequal chest expansion b. Increased tactile fremitus c. Atrophied neck and trapezius muscles d. Anteroposterior-to-transverse diameter ratio of 1:1

ANS: D An anteroposterior-to-transverse diameter ratio of 1:1 or barrel chest is observed in individuals with COPD because of hyperinflation of the lungs. The ribs are more horizontal, and the chest appears as if held in continuous inspiration. Neck muscles are hypertrophied from aiding in forced respiration. Chest expansion may be decreased but symmetric. Decreased tactile fremitus occurs from decreased transmission of vibrations.

During a symptom analysis, a patient describes his productive cough and states his sputum is thick and yellow. Based on these data, the nurse suspects which factor as the cause of these symptoms? a. Virus b. Allergy c. Fungus d. Bacteria

ANS: D Feedback A A virus usually produces a nonproductive cough. B An allergy usually produces clear sputum. C A fungus usually produces few symptoms. The sputum used to diagnose the fungus is obtained from tracheal aspiration rather than the patient coughing up the sputum. D Bacteria usually produce sputum that is yellow or green in color.

A patient is admitted to the emergency department with a tracheal obstruction. What sound does the nurse expect to hear as this patient breathes? a. Dull sounds on percussion b. Soft, muffled rhonchi heard over the trachea c. Bubbling or rasping sounds heard over the trachea d. High-pitched sounds on inspiration and exhalation

ANS: D Feedback A Dull sounds on percussion occur with pneumonia, pleural effusion, or atelectasis. B Soft, muffled rhonchi heard over the trachea is not a description of stridor. C Bubbling or rasping sounds heard over the trachea is not a description of stridor. D High-pitched sounds on inspiration and exhalation are consistent with stridor.

A nurse in the emergency department is assessing a patient with a moderate left pneumothorax. What does this nurse expect to find during the respiratory examination? a. Increased fremitus over the left chest b. Tracheal deviation to the left side c. Hyporesonant percussion tones over the left chest d. Distant to absent breath sounds over the left chest

ANS: D Feedback A Increased fremitus occurs over lung consolidation as in lobar pneumonia or tumor. B If this patient had a tension pneumothorax, the trachea would deviate to the right. C Hyperresonant percussion tones are heard when the lung is overinflated as in emphysema. D The air separating the lung from the chest where the nurse is auscultating creates distant to absent breath sounds.

Absent breath sounds

atelectasis

During a history, a nurse notices a patient is short of breath, is using pursed-lip breathing, and maintains a tripod position. Based on these data, what abnormal finding should the nurse expect to find during the examination? a. Increased tactile fremitus b. Inspiratory and expiratory wheezing c. Tracheal deviation d. An increased anteroposterior diameter

ANS: D Feedback A Increased tactile fremitus occurs when vibrations are enhanced and is associated with consolidation that may occur in pneumonia or tumor. B Inspiratory and expiratory wheezing is associated with asthma. C Tracheal deviation is associated with tension pneumothorax. D An increased anteroposterior diameter is consistent with emphysema.

Where does a nurse expect to hear bronchovesicular lung sounds in a healthy adult? a. In the lower lobes b. Over the trachea c. In the apices of the lungs d. Near the sternal border

ANS: D Feedback A Vesicular breath sounds are normally heard in the lower lobes. B Bronchial sounds are normally heard over the trachea. C Vesicular breath sounds are normally heard in the apices of the lungs. D Bronchovesicular breath sounds are normally heard over the central area of the anterior thorax around the sternal border.

The nurse is assessing the lungs of an older adult. Which of these changes are normal in the respiratory system of the older adult? a. Severe dyspnea is experienced on exertion, resulting from changes in the lungs. b. Respiratory muscle strength increases to compensate for a decreased vital capacity. c. Decrease in small airway closure occurs, leading to problems with atelectasis. d. Lungs are less elastic and distensible, which decreases their ability to collapse and recoil.

ANS: D In the aging adult, the lungs are less elastic and distensible, which decreases their ability to collapse and recoil. Vital capacity is decreased, and a loss of intra-alveolar septa occurs, causing less surface area for gas exchange. The lung bases become less ventilated, and the older person is at risk for dyspnea with exertion beyond his or her usual workload.

When inspecting the anterior chest of an adult, the nurse should include which assessment? a. Diaphragmatic excursion b. Symmetric chest expansion c. Presence of breath sounds d. Shape and configuration of the chest wall

ANS: D Inspection of the anterior chest includes shape and configuration of the chest wall; assessment of the patient's level of consciousness and the patient's skin color and condition; quality of respirations; presence or absence of retraction and bulging of the intercostal spaces; and use of accessory muscles. Symmetric chest expansion is assessed by palpation. Diaphragmatic excursion is assessed by percussion of the posterior chest. Breath sounds are assessed by auscultation.

During a morning assessment, the nurse notices that the patient's sputum is frothy and pink. Which condition could this finding indicate? a. Croup b. Tuberculosis c. Viral infection d. Pulmonary edema

ANS: D Sputum, alone, is not diagnostic, but some conditions have characteristic sputum production. Pink, frothy sputum indicates pulmonary edema or it may be a side effect of sympathomimetic medications. Croup is associated with a barking cough, not sputum production. Tuberculosis may produce rust-colored sputum. Viral infections may produce white or clear mucoid sputum.

FEV (Forced expiratory volume)

Amount of air exhaled in first second of forced vital capacity

RV (Residual volume)

Amount of air remaining in lungs after forced expiration

FVC (Forved vital capacity)

Amount of air that can be quickly and forcefully exhaled after maximum inspiration

A female client is suspected of having a pulmonary embolus. A nurse assesses the client, knowing that which of the following is a common clinical manifestation of pulmonary embolism? a. Dyspnea b. Bradypnea c. Bradycardia d. Decreased respiratory

Answer A. The common clinical manifestations of pulmonary embolism are tachypnea, tachycardia, dyspnea, and chest pain

2. Nurse Kim is caring for a client with a pneumothorax and who has had a chest tube inserted notes continuous gentle bubbling in the suction control chamber. What action is appropriate? a. Do nothing, because this is an expected finding. b. Immediately clamp the chest tube and notify the physician. c. Check for an air leak because the bubbling should be intermittent. d. Increase the suction pressure so that bubbling becomes vigorous.

Answer A. Continuous gentle bubbling should be noted in the suction control chamber. Option B is incorrect. Chest tubes should only be clamped to check for an air leak or when changing drainage devices (according to agency policy). Option C is incorrect. Bubbling should be continuous and not intermittent. Option D is incorrect because bubbling should be gentle. Increasing the suction pressure only increases the rate of evaporation of water in the drainage system.

An unconscious male client is admitted to an emergency room. Arterial blood gas measurements reveal a pH of 7.30, a low bicarbonate level, a normal carbon dioxide level, a normal oxygen level, and an elevated potassium level. These results indicate the presence of: a. Metabolic acidosis b. Respiratory acidosis c. Overcompensated respiratory acidosis d. Combined respiratory and metabolic acidosis

Answer A. In an acidotic condition, the pH would be low, indicating the acidosis. In addition, a low bicarbonate level along with the low pH would indicate a metabolic state. Therefore, options B, C, and D are incorrect.

A male client has been admitted with chest trauma after a motor vehicle accident and has undergone subsequent intubation. A nurse checks the client when the high-pressure alarm on the ventilator sounds, and notes that the client has absence of breathe sounds in right upper lobe of the lung. The nurse immediately assesses for other signs of: a. Right pneumothorax b. Pulmonary embolism c. Displaced endotracheal tube d. Acute respiratory distress syndrome

Answer A. Pneumothorax is characterized by restlessness, tachycardia, dyspnea, pain with respiration, asymmetrical chest expansion, and diminished or absent breath sounds on the affected side. Pneumothorax can cause increased airway pressure because of resistance to lung inflation. Acute respiratory distress syndrome and pulmonary embolism are not characterized by absent breath sounds. An endotracheal tube that is inserted too far can cause absent breath sounds, but the lack of breath sounds most likely would be on the left side because of the degree of curvature of the right and left main stem bronchi.

A nurse is teaching a male client with chronic respiratory failure how to use a metered-dose inhaler correctly. The nurse instructs the client to: a. Inhale quickly b. Inhale through the nose c. Hold the breath after inhalation d. Take two inhalations during one breath

Answer C. Instructions for using a metered-dose inhaler include - shaking the canister, - holding it right side up, - inhaling slowly and evenly through the mouth, - delivering one spray per breath, - and holding the breath after inhalation.

A nurse teaches a male client about the use of a respiratory inhaler. Which action by the client indicates a need for further teaching? a. Inhales the mist and quickly exhales b. Removes the cap and shakes the inhaler well before use c. Presses the canister down with the finger as he breathes in d. Waits 1 to 2 minutes between puffs if more than one puff has been prescribed

Answer A. The client should be instructed to hold his or her breath for at least 10 to 15 seconds before exhaling the mist. Options B, C, and D are accurate instructions regarding the use of the inhaler.

A nurse is assessing a male client with chronic airflow limitations and notes that the client has a "barrel chest." The nurse interprets that this client has which of the following forms of chronic airflow limitations? a. Emphysema b. Bronchial asthma c. Chronic obstructive bronchitis d. Bronchial asthma and bronchitis

Answer A. The client with emphysema has hyperinflation of the alveoli and flattening of the diaphragm. These lead to increased anteroposterior diameter, referred to as "barrel chest." The client also has dyspnea with prolonged expiration and has hyperresonant lungs to percussion.

A nurse is caring for a male client immediately after removal of the endotracheal tube. The nurse reports which of the following signs immediately if experienced by the client? a. Stridor b. Occasional pink-tinged sputum c. A few basilar lung crackles on the right d. Respiratory rate of 24 breaths/min

Answer A. The nurse reports stridor to the physician immediately. This is a high-pitched, coarse sound that is heard with the stethoscope over the trachea. Stridor indicates airway edema and places the client at risk for airway obstruction

4. The nurse caring for a male client with a chest tube turns the client to the side, and the chest tube accidentally disconnects. The initial nursing action is to: a. Call the physician. b. Place the tube in a bottle of sterile water. c. Immediately replace the chest tube system. d. Place the sterile dressing over the disconnection site.

Answer B. If the chest drainage system is disconnected, the end of the tube is placed in a bottle of sterile water held below the level of the chest. The system is replaced if it breaks or cracks or if the collection chamber is full. Placing a sterile dressing over the disconnection site will not prevent complications resulting from the disconnection. The physician may need to be notified, but this is not the initial action.

A nurse is caring for a male client hospitalized with acute exacerbation of chronic obstructive pulmonary disease. Which of the following would the nurse expect to note on assessment of this client? a. Hypocapnia b. A hyperinflated chest noted on the chest x-ray c. Increase oxygen saturation with exercise d. A widened diaphragm noted on the chest x-ray

Answer B. Clinical manifestations of chronic obstructive pulmonary disease (COPD) include hypoxemia, - hypercapnia, - dyspnea on exertion and at rest - oxygen desaturation with exercise - and the use of accessory muscles of respiration. Chest x-rays reveal a hyperinflated chest and a flattened diaphragm if the disease is advanced.

A nurse is caring for a male client with emphysema who is receiving oxygen. The nurse assesses the oxygen flow rate to ensure that it does not exceed: a. 1 L/min b. 2 L/min c. 6 L/min d. 10 L/min

Answer B. Oxygen is used cautiously and should not exceed 2 L/min. Because of the long-standing hypercapnia that occurs in emphysema, the respiratory drive is triggered by low oxygen levels rather than increased carbon dioxide levels, as is the case in a normal respiratory system.

A nurse is caring for a female client diagnosed with tuberculosis. Which assessment, if made by the nurse, is inconsistent with the usual clinical presentation of tuberculosis and may indicate the development of a concurrent problem? a. Cough b. High-grade fever c. Chills and night sweats d. Anorexia and weight loss

Answer B. The client with tuberculosis USUALLY experiences cough (productive or nonproductive), fatigue, anorexia, weight loss, dyspnea, hemoptysis, chest discomfort or pain, chills and sweats (which may occur at night), and a low-grade fever

An emergency room nurse is assessing a female client who has sustained a blunt injury to the chest wall. Which of these signs would indicate the presence of a pneumothorax in this client? a. A low respiratory b. Diminished breathe sounds c. The presence of a barrel chest d. A sucking sound at the site of injury

Answer B. This client has sustained a blunt or a closed chest injury. Basic symptoms of a closed pneumothorax are shortness of breath and chest pain. A larger pneumothorax may cause tachypnea, cyanosis, diminished breath sounds, and subcutaneous emphysema. Hyperresonance also may occur on the affected side. A sucking sound at the site of injury would be noted with an open chest injury.

Nurse Hannah is preparing to obtain a sputum specimen from a client. Which of the following nursing actions will facilitate obtaining the specimen? a. Limiting fluids b. Having the clients take three deep breaths c. Asking the client to split into the collection container d. Asking the client to obtain the specimen after eating

Answer B. To obtain a sputum specimen, the client should rinse the mouth to reduce contamination, breathe deeply, and then cough into a sputum specimen container. The client should be encouraged to cough and not spit so as to obtain sputum. Sputum can be thinned by fluids or by a respiratory treatment such as inhalation of nebulized saline or water. The optimal time to obtain a specimen is on arising in the morning

A nurse is caring for a female client after a bronchoscope and biopsy. Which of the following signs, if noted in the client, should be reported immediately to the physicians? a. Dry cough b. Hematuria c. Bronchospasm d. Blood-streaked sputum

Answer C. If a biopsy was performed during a bronchoscopy, blood-streaked sputum is expected for several hours. Frank blood indicates hemorrhage. A dry cough may be expected. The client should be assessed for signs of complications, which would include cyanosis, dyspnea, stridor, bronchospasm, hemoptysis, hypotension, tachycardia, and dysrhythmias. Hematuria is unrelated to this procedure.

A nurse is suctioning fluids from a female client through an endotracheal tube. During the suctioning procedure, the nurse notes on the monitor that the heart rate is decreasing. Which of the following is the appropriate nursing intervention? a. Continue to suction. b. Notify the physician immediately. c. Stop the procedure and reoxygenate the client. d. Ensure that the suction is limited to 15 seconds.

Answer C. During suctioning, the nurse should monitor the client closely for side effects, including hypoxemia, cardiac irregularities such as a decrease in heart rate resulting from vagal stimulation, mucosal trauma, hypotension, and paroxysmal coughing. If side effects develop, especially cardiac irregularities, the procedure is stopped and the client is reoxygenated.

A nurse is suctioning fluids from a male client via a tracheostomy tube. When suctioning, the nurse must limit the suctioning time to a maximum of: a. 1 minute b. 5 seconds c. 10 seconds d. 30 seconds

Answer C. Hypoxemia can be caused by prolonged suctioning, which stimulates the pacemaker cells in the heart. A vasovagal response may occur, causing bradycardia. The nurse must preoxygenate the client before suctioning and limit the suctioning pass to 10 seconds.

A female client has just returned to a nursing unit following bronchoscopy. A nurse would implement which of the following nursing interventions for this client? a. Administering atropine intravenously b. Administering small doses of midazolam (Versed) c. Encouraging additional fluids for the next 24 hours d. Ensuring the return of the gag reflex before offering food or fluids

Answer D. After bronchoscopy, the nurse keeps the client on NPO status until the gag reflex returns because the preoperative sedation and local anesthesia impair swallowing and the protective laryngeal reflexes for a number of hours. Additional fluids are unnecessary because no contrast dye is used that would need flushing from the system. Atropine and midazolam would be administered before the procedure, not after.

A community health nurse is conducting an educational session with community members regarding tuberculosis. The nurse tells the group that one of the first symptoms associated with tuberculosis is: a. Dyspnea b. Chest pain c. A bloody, productive cough d. A cough with the expectoration of mucoid sputum

Answer D. One of the first pulmonary symptoms is a slight cough with the expectoration of mucoid sputum. Options A, B, and C are late symptoms and signify cavitation and extensive lung involvement.

A nurse instructs a female client to use the pursed-lip method of breathing and the client asks the nurse about the purpose of this type of breathing. The nurse responds, knowing that the primary purpose of pursed-lip breathing is to: a. Promote oxygen intake. b. Strengthen the diaphragm. c. Strengthen the intercostal muscles. d. Promote carbon dioxide elimination.

Answer D. Pursed-lip breathing facilitates maximal expiration for clients with obstructive lung disease. This type of breathing allows better expiration by increasing airway pressure that keeps air passages open during exhalation. Options A, B, and C are not the purposes of this type of breathing.

A nurse is assessing the respiratory status of a male client who has suffered a fractured rib. The nurse would expect to note which of the following? a. Slow deep respirations b. Rapid deep respirations c. Paradoxical respirations d. Pain, especially with inspiration

Answer D. Rib fractures are a common injury, especially in the older client, and result from a blunt injury or a fall. Typical signs and Sx include - pain and tenderness localized at the fracture site and exacerbated by inspiration and palpation - shallow respirations - splinting or guarding the chest protectively to minimize chest movement, and possible bruising at the fracture site. Paradoxical respirations are seen with flail chest.

A nurse is assessing a female client with multiple trauma who is at risk for developing acute respiratory distress syndrome. The nurse assesses for which earliest sign of acute respiratory distress syndrome? a. Bilateral wheezing b. Inspiratory crackles c. Intercostal retractions d. Increased respiratory rate

Answer D. The earliest detectable sign of acute respiratory distress syndrome is an increased respiratory rate, which can begin from 1 to 96 hours after the initial insult to the body. T his is followed by increasing dyspnea, air hunger, retraction of accessory muscles, and cyanosis. Breath sounds may be clear or consist of fine inspiratory crackles or diffuse coarse crackles.

Presence of overdistended and non-functional alveoli is a condition called: a. Bronchitis b. Emphysema c. Empyema d. Atelectasis

Answer: B. An overdistended and non-functional alveoli is a condition called emphysema. Atelectasis is the collapse of a part or the whole lung. Empyema is the presence of pus in the lung.

Identify what is included during the assessment phase of the nursing process for a cardiopulmonary focus.

Assessment • In-depth history of the client's normal and present cardiopulmonary function • Past impairments in circulatory or respiratory functioning • Patient history including a review of drug, food, and other allergies • Physical examination of the client's cardiopulmonary status reveals the extent of existing signs and symptoms. • Use PQRST for pain / HPI for other symptoms • Review of laboratory and diagnostic test results

To detect early signs or symptoms of inadequate oxygenation, the nurse would examine the patient for A. Dyspnea and hypotension B. Apprehension and restlessness C. Cyanosis and cool, clammy skin D. Increased urine output and diaphoresis

B. Apprehension and restlessness

What covers the larynx during swallowing A Trachea B Epiglotis C Turbinates D Parietal pleura

B Epiglottis

A pulse oximetry monitor indicates that the patient has a drop in Sp02 fom 95% to 85% over several hours. What is the first action the nurse should take A Order stat ABGs to confirm the Sp02 with a Sa02 B Start oxygen administration by nasal cannula at 2L/min C Check the position of the probe on the finger or earlobe D Notify the health care provider of the change in baseline Pa02

B Start oxygen administration by nasal cannula at 2L/min

When does the nurse record the presence of an increased anteroposterior (AP) diameter of the chest? A There is a prominent protrusion of the sternum B The width of the chest is equal to the depth of the chest C There is equal but diminished movement of the two sides of the chest D The patient cannot fully expand the lungs because of kyphosis of the spine

B The width of the chest is equal to the depth of the chest

What keep alveoli from collapsing? A carina b surfactant c empyema d thoracic cage

B surfactant

When assessing a patient's sleep-rest pattern related to respiratory health, what should the nurse ask the patient (select all that apply.)? a. Do you sleep more than 8 hours per night? b. Do you awaken abruptly during the night? c. Do you often need to urinate during the night? d. Do you need to sleep with the head elevated? e. Do you toss and turn when trying to fall asleep?

B, D, E A patient with obstructive sleep apnea may have insomnia, abrupt awakenings, or both. Patients with cardiovascular disease (e.g., heart failure that may affect respiratory health) may need to sleep with the head elevated on several pillows (orthopnea). Sleeping more than 8 hours per night or needing to urinate during the night is not indicative of impaired respiratory health.

A 92 year old female patient is being admitted to the emergency department with severe shortness of breath. Being aware of the patients condition, what approach should the nurse use to assess the patient's lungs (select all that apply) A apex to base B base to apex C Lateral sequence D anterior then posterior E Posterior then anterior

B, E

Which patient is exhibiting an early clinical manifestation of hypoxemia? a. A 94-yr-old patient who has renal insufficiency, anemia, and decreased urine output b. A 72-yr-old patient who has four new premature ventricular contractions per minute c. A 67-yr-old patient who has dyspnea while resting in the bed or in a reclining chair d.. A 48-yr-old patient who is intoxicated and acutely disoriented to time and place

B. A 72-yr-old patient who has four new premature ventricular contractions per minute Early clinical manifestations of hypoxemia include dysrhythmias (e.g., premature ventricular contractions), unexplained decreased level of consciousness (e.g., disorientation), dyspnea on exertion, and unexplained decreased urine output.

The patient is hospitalized with pneumonia. Which diagnostic test should be used to measure the efficiency of gas exchange in the lung and tissue oxygenation? a. Pulmonary function tests b. Arterial blood gases c. Thoracentesis d. Bronchoscopy

B. Arterial blood gases Arterial blood gases are used to assess the efficiency of gas exchange in the lung and tissue oxygenation as is pulse oximetry. Thoracentesis is used to obtain specimens for diagnostic evaluation, remove pleural fluid, or instill medication into the pleural space. Bronchoscopy is used for diagnostic purposes, to obtain biopsy specimens, and to assess changes resulting from treatment. Pulmonary function tests measure lung volumes and airflow to diagnose pulmonary disease, monitor disease progression, evaluate disability, and evaluate response to bronchodilators.

When auscultating the chest of an older patient in respiratory distress, it is best to A. Begin listening at the apices B. Begin listening at the lung bases C. Begin listening on the anterior chest D. Ask the patient to breathe through the nose with the mouth closed.

B. Begin listening at the lung bases

The patient is calling the clinic with a cough. What assessment should be made first before the nurse advises the patient? a. Weight loss, activity tolerance, orthopnea b. Cough sound, sputum production, pattern c. Smoking status, medications, residence location d. Frequency, family history, hematemesis

B. Cough sound, sputum production, pattern The sound of the cough, sputum production and description, and the pattern of the cough's occurrence (including acute or chronic) and what its occurrence is related to are the first assessments to be made to determine the severity. Frequency of the cough will not provide a lot of information. Family history can help to determine a genetic cause of the cough. Hematemesis is vomiting blood and not as important as hemoptysis. Smoking is an important risk factor for chronic obstructive pulmonary disease, and lung cancer and may cause a cough. Medications may or may not contribute to a cough as does residence location. Weight loss, activity intolerance, and orthopnea may be related to respiratory or cardiac problems, but are not a

After swallowing, a 73-yr-old patient is coughing and has a wet voice. What changes of aging could be contributing to this abnormal finding? a. Decreased number of functional alveoli b. Decreased respiratory defense mechanisms c. Increased calcification of costal cartilage d. Decreased response to hypercapnia

B. Decreased respiratory defense mechanisms These manifestations are associated with aspiration, which more easily occur in the right lung as the right mainstem bronchus is shorter, wider, and straighter than the left mainstem bronchus. Aspiration occurs more easily in the older patient related to decreased respiratory defense mechanisms (e.g., decreases in immunity, ciliary function, cough force, sensation in pharynx). Changes of aging include a decreased response to hypercapnia, decreased number of functional alveoli, and increased calcification of costal cartilage, but these do not increase the risk of aspiration.

Wheezes

Bronchoconstriction

Which respiratory defense mechanism is most impaired by smoking? A Cough reflux B Filtration of air C Mucociliary clearance D Reflex bronchoconstriction

C Mucociliary clearance

The abnormal assessment findings of dullness and hyperresonance are found with which assessment technique? A Inspection B Palpation C Percussion D Auscultation

C Percussion

What is a primary nursing responsibility after obtaining a blood specimen for ABGs? A ad heparin to the blood specimen B apply pressure to the puncture site for 2 full minutes C Take the specimen immediately to the laboratory in an iced container D Avoid any changes in oxygen intervention for 15 minutes following the procduere.

C Take the specimen immediately to the laboratory in an iced container

What accurately describes the alveolar sacs? A Line the lung pleura B warm and moisturize inhaled air C Terminal structures of the respiratory tract D Contain dead air that is not available for gas exhange

C Terminal structures of the respiratory tract

Why does a patient's respiratory rate increase when there is an excess of carbon dioxide in the blood? A co2 displaces oxygen on hemoglobin, leading to a decreased Pa02 B co2 causes an increase in the amount of hydrogen ions available in the body C co2 combines with water to form carbonic acid, which lowers the pH of cerebrospinal fluid D co2 directly stiulates chemoreceptors in the medulla to increase respiratory rate and volume

C co2 combines with water to form carbonic acid, which lowers the pH of cerebrospinal fluid

Pulse oximetry may not be a reliable indicator of oxygen saturation in which patient? A patient with a fever B Patient who is anesthetied C patient in hypovolemic shock D patient receiving oxygen therapy

C patient in hypovolemic shock

How does the nurse assess the patient's chest expansion? A put the palms of the hands against the chest wall B put the index fingers on either side of the trachea C place the thumbs at the midline of the lower chest D place one hand on the lower anterior chest and one hand on the upper abdomen

C place the thumbs at the midline of the lower chest

Palpation is the assessment technique used to find which abnormal assessment findings (select all that apply) A Stridor B Finger clubbing C Tracheal deviation D Limited chest expansion E Increased tactile fremitus F Use of accessory muscles

C, D, E

In assessment of the patient with acute respiratory distress, what should the nurse expect to observe (select all that apply.)? a. Cyanosis b. Kussmaul respirations c. Tripod position Correct d. Increased AP diameter e. Accessory muscle use

C, E Tripod position and accessory muscle use indicate moderate to severe respiratory distress. Cyanosis may be related to anemia, decreased oxygen transfer in the lungs, or decreased cardiac output. Therefore, it is a nonspecific and unreliable indicator of only respiratory distress. Kussmaul respirations occur when the patient is in metabolic acidosis to increase CO2 excretion. Increased AP diameter occurs with lung hyperinflation from chronic obstructive pulmonary disease, cystic fibrosis, or with advanced age.

A 67-yr-old male patient had a right total knee replacement 2 days ago. Upon auscultation of the patient's posterior chest, the nurse detects discontinuous, high-pitched breath sounds just before the end of inspiration in the lower portion of both lungs. Which statement most appropriately reflects how the nurse should document the breath sounds? A. "Diminished breath sounds in the bases of both lungs." B. "Expiratory wheezing scattered throughout the lung fields." C. "Fine crackles posterior right and left lower lung fields." D. "Bibasilar wheezes present on inspiration."

C. "Fine crackles posterior right and left lower lung fields." Fine crackles are described as a series of short-duration, discontinuous, high-pitched sounds heard just before the end of inspiration.

The nurse is caring for a patient with chronic obstructive pulmonary disorder (COPD) and pneumonia who has an order for arterial blood gases to be drawn. What is the minimum length of time the nurse should plan to hold pressure on the puncture site? a. 15 minutes b. 10 minutes c. 5 minutes Correct d. 2 minutes

C. 5 minutes After obtaining blood for an arterial blood gas measurement, the nurse should hold pressure on the puncture site for 5 minutes by the clock to be sure that bleeding has stopped. An artery is an elastic vessel under much higher pressure than veins, and significant blood loss or hematoma formation could occur if the time is insufficient.

The nurse is caring for a patient who had abdominal surgery yesterday. Today the patient's lung sounds in the lower lobes are diminished. The nurse knows this could be related to the occurrence of a. pneumonia. b. pleural effusion. c. atelectasis. d. pain.

C. Atelectasis Postoperatively, there is an increased risk for atelectasis from anesthesia as well as restricted breathing from pain. Without deep breathing to stretch the alveoli, surfactant secretion to hold the alveoli open is not promoted. Pneumonia will occur later after surgery. Pleural effusion occurs because of blockage of lymphatic drainage or an imbalance between intravascular and oncotic fluid pressures, which is not expected in this case.

The nurse, when auscultating the lower lungs of the patient, hears these breath sounds. How should the nurse document these sounds? a. Vesicular b. Stridor c. Coarse crackles d. Bronchovesicular

C. Coarse Crackles Coarse crackles are a series of long-duration, discontinuous, low-pitched sounds caused by air passing through an airway intermittently occluded by mucus, an unstable bronchial wall, or a fold of mucosa. Coarse crackles are evident on inspiration and at times expiration. Stridor is a continuous crowing sound of constant pitch from partial obstruction of larynx or trachea. Vesicular sounds are relatively soft, low-pitched, gentle, rustling sounds. They are heard over all lung areas except the major bronchi. Bronchovesicular sounds are normal sounds heard anteriorly over the mainstem bronchi on either side of the sternum and posteriorly between the scapulae with a medium pitch and intensity.

What should the nurse inspect when assessing a patient with shortness of breath for evidence of long-standing hypoxemia? a. Respiratory pattern b. Spinal curvatures c. Fingernails d. Chest excursion

C. Fingernails Clubbing, a sign of long-standing hypoxemia, is evidenced by an increase in the angle between the base of the nail and fingernail to 180 degrees or more, usually accompanied by an increase in the depth, bulk, and sponginess of the end of the finger.

During the respiratory assessment of an older adult, the nurse would expect to find (select all that apply) A. a vigorous reflex cough. B. Increased chest expansion C. Increased residual volume D. Diminished lung sounds at base of lungs E. Increase anteroposterior (AP) chest diameter

C. Increased residual volume D. Diminished lung sounds at base of lungs E. Increase anteroposterior (AP) chest diameter

The patient's arterial blood gas results show the PaO2 at 65 mmHg and SaO2 at 80%. What early manifestations should the nurse expect to observe in this patient? a. Unexplained confusion, dyspnea at rest, hypotension, and diaphoresis b. Combativeness, retractions with breathing, cyanosis, and decreased output c. Restlessness, tachypnea, tachycardia, and diaphoresis d. Coma, accessory muscle use, cool and clammy skin, and unexplained fatigue

C. Restlessness, tachypnea, tachycardia, and diaphoresis Correct With inadequate oxygenation, early manifestations include restlessness, tachypnea, tachycardia, and diaphoresis, decreased urinary output, and unexplained fatigue. The unexplained confusion, dyspnea at rest, hypotension, and diaphoresis; combativeness, retractions with breathing, cyanosis, and decreased urinary output; coma, accessory muscle use, cool and clammy skin, and unexplained fatigue occur as later manifestations of inadequate oxygenation.

Finger clubbing

Caused by chronic hypoxemia

Elimination

Constipation, incontinence

Wheezes

Continuous high-pitched squeaking or musical sound caused by rapid vibration of bronchial walls. First evident on expiration but possibly evident on inspiration as obstruction of airway increases. Possibly audible without stethoscope

Pleural friction rub

Creaking or grating sound from roughened, inflamed pleural surfaces rubbing together. Evident during inspiration

A 73 year old patient has an Sp02 of 70%. What other assessment should the nurse consider before making a judgement about the adequacy of the patient's oxygenation? A What the oxygenation status is with a stress test B Trend and rate of development of the hyperkalemis C Comparison of patient's Sp02 values with the normal values D Comparison of patient's current vital signs with normal vital signs

D Comparison of patient's current vital signs with normal vital signs

A patient's ABGs include a Pa02 of 88mm Hg and a PaC)2 of 38mm Hg, and mixed venous blood gases include a Pv02 of 40 mm Hg and PvCo2 of 46 mm Hg. What do these findings indicate? A Impaired cardiac output B Unstable hemodynamics C Inadequate delivery of oxygen to the tissues D Normal capillary oxygen-carbon dioxide exchange

D Normal capillary oxygen-carbon dioxide exchange

The patient is admitted with pneumonia, and the nurse hears a grating sound when she assesses the patient. How should the nurse document this sound? A Stridor B Bronchophony C Course crackles D Pleural friction rub

D Pleural friction rub

The health care provider orders a pulmonary angiogram for a patient admitted with dyspnea and hemoptysis. For which problem is this test most commonly used as a diagnostic measure? A Tuberculosis B Cancer of the lung C Airway obstruction D Pulmonary embolism

D Pulmonary embolism

Which values indicate a need for the use of continuous oxygen therapy? A Sp02 of 92%, Pa02 of 65 mm Hg B Sp02 of 95%, Pa02 of 70 mm Hg C Sp02 of 90%, Pa02 of 60 mm Hg D Sp02 of 88%, Pa02 of 55 mm Hg

D Sp02 of 88%, Pa02 of 55 mm Hg

A nurse has been caring for a patient with TB and has a TB skin test performed. When is the nurse considered infected? A There is no redness or induration at the injection site B There is an induration of only 5 mm at teh injection site C A negative skin test is followed by a negative chest x-ray D Testing causes a 10-mm red, indurated area at the injection site

D Testing causes a 10-mm red, indurated area at the injection site

The nurse is interpreting a tuberculin skin test (TST) for a 58-yr-old female patient with end-stage renal disease secondary to diabetes mellitus. Which finding would indicate a positive reaction? a. Acid-fast bacilli cultured at the injection site b. 15-mm area of redness at the TST injection site c. Wheal formed immediately after intradermal injection d. 11-mm area of induration at the TST injection site

D. 1-mm area of induration at the TST injection site An area of induration 10 mm or larger would be a positive reaction in a person with end-stage renal disease. Reddened, flat areas do not indicate a positive reaction. A wheal appears when the TST is administered that indicates correct administration of the intradermal antigen. Presence of acid-fast bacilli in the sputum indicates active tuberculosis.

The patient with Parkinson's disease has a pulse oximetry reading of 72%, but he is not displaying any other signs of decreased oxygenation. What is most likely contributing to his low SpO2 level? A. Anemia B. Thick acrylic nails C. Dark skin color D. Artifact

D. Artifact Motion is the most likely cause of the low SpO2 for this patient with Parkinson's disease. Anemia, dark skin color, and thick acrylic nails as well as low perfusion, bright fluorescent lights, and intravascular dyes may also cause an inaccurate pulse oximetry result. There is no mention of these or reason to suspect these in this question.

The nurse is obtaining a focused respiratory assessment of a 44-yr-old female patient who is in severe respiratory distress 2 days after abdominal surgery. What is most important for the nurse to assess? a. Palpation of the chest bilaterally for tactile fremitus b. Inspection for anterior and posterior chest expansion c. Percussion of anterior and posterior chest wall d. Auscultation of bilateral breath sounds

D. Auscultation of bilateral breath sounds Important assessments obtained during a focused respiratory assessment include auscultation of lung (breath) sounds. Assessment of tactile fremitus has limited value in acute respiratory distress. It is not necessary to assess for both anterior and posterior chest expansion. Percussion of the chest wall is not essential in a focused respiratory assessment.

A frail 82-yr-old female patient develops sudden shortness of breath while sitting in a chair. What location on the chest should the nurse begin auscultation of the lung fields? a. Apices of the posterior lung fields b. Midaxillary on the left side of the chest c. Anterior chest area above the breasts d. Bases of the posterior chest area

D. Bases of the posterior chest area Baseline data with the most information is best obtained by auscultation of the posterior chest, especially in female patients because of breast tissue interfering with the assessment or if the patient may tire easily (e.g., shortness of breath, dyspnea, weakness, fatigue). Usually auscultation proceeds from the lung apices to the bases unless it is possible the patient will tire easily. In this case, the nurse should start at the bases.

Which assessment finding of the respiratory system does the nurse interpret as abnormal A. Inspiratory chest expansion of 1 inch B. Symmetric chest expansion and contraction C. Resonance (to percussion) over the lung bases D. Bronchial breath sounds in the lower lung fields

D. Bronchial breath sounds in the lower lung fields

A patient with a respiratory condition asks, "How does air get into my lungs?" The nurse bases her answer on knowledge that air moves into the lungs because of A. Increased CO2 and decreased 02 in the blood B. Contraction of the accessory abdominal muscles C. Stimulation of the respiratory muscles by the chemoreceptors. D. Decrease in intrathoracic pressure relative to pressure at the airway

D. Decrease in intrathoracic pressure relative to pressure at the airway

When the patient is experiencing metabolic acidosis secondary to type 1 diabetes mellitus, what physiologic response should the nurse expect to assess in the patient? a. Increased urination b. Decreased heart rate c. Vomiting d. Increased respiratory rate

D. Increased respiratory rate When a patient with type 1 diabetes has hyperglycemia and ketonemia causing metabolic acidosis, the physiologic response is to increase the respiratory rate and tidal volume to blow off the excess CO2. Vomiting and increased urination may occur with hyperglycemia, but not as physiologic responses to metabolic acidosis. The heart rate will increase.

A patient with recurrent shortness of breath has just had a bronchoscopy. What is a priority nursing action immediately after the procedure? A. Monitor and manage the patient's level of pain. B. Assess the patient's heart rate and blood pressure. C. Assess the patient's level of consciousness. D. Monitor the patient for laryngeal edema.

D. Monitor the patient for laryngeal edema. Priorities for assessment are the patient's airway and breathing, both of which may be compromised after bronchoscopy by laryngeal edema. These assessment parameters supersede the importance of loss of consciousness (LOC), pain, heart rate, and blood pressure, although the nurse should also be assessing these.

After assisting at the bedside with a thoracentesis, the nurse should continue to assess the patient for signs and symptoms of what? a. Pulmonary edema b. Respiratory acidosis c. Bronchospasm d. Pneumothorax

D. Pneumothorax Because thoracentesis involves the introduction of a catheter into the pleural space, there is a risk of pneumothorax. Thoracentesis does not carry a significant potential for causing bronchospasm, pulmonary edema, or respiratory acidosis.

During the assessment in the emergency department, the nurse is palpating the patient's chest. Which finding is a medical emergency? A. Diminished chest movement B. Decreased anteroposterior (AP) diameter C. Increased tactile fremitus D. Tracheal deviation to the left

D. Tracheal deviation to the left Tracheal deviation is a medical emergency when it is caused by a tension pneumothorax. Tactile fremitus increases with pneumonia or pulmonary edema and decreases in pleural effusion or lung hyperinflation. Diminished chest movement occurs with barrel chest, restrictive disease, and neuromuscular disease.

Activity-exercise

Decreased exercise or activity tolerance, dyspnea on rest or exertion, sedentary habits

Nutritional-metabolic

Decreased fluid intake, anorexia and rapid weight loss, obesity

Coping-stress tolerance

Dyspnea-anxiety-dyspnea cycle, poor coping with stress of chronic respiratory problems

In reviewing the patient's record, the nurse notes that the patient has air in the subcutaneous tissue. The nurse validates that this patient has crepitus with which finding? a. Asymmetric expansion of the chest wall on inhalation b. Increased transmission of vocal vibrations on auscultation c. Crackling sensation under the skin of the chest on palpation d. Coarse grating sounds heard over the mediastinum on inspiration ANS: C

Feedback A Asymmetric chest expansion occurs with rib fracture or chest wall injury. B Increased vocal fremitus occurs with lung consolidation. C A crackling sensation is the finding when crepitus is present. D Coarse grating sounds heard over the mediastinum on inspiration does not validate crepitus.

What does FIO2 stand for?

Fraction of Inspired oxygen concentration

What does central cyanosis indicate?

Hypoexmia

Atelectasis

Impression: No change unless involves entire segment, lobe Palpation: I small, no change. If large, decrease mevement, decrease fremitus Percussion: Dull over affected area Auscultation: Fine crackles (may disappear with deep breaths)

Self perception, self concept

Inability to maintain lifestyle, altered self-esteem

COPD

Inspection: Barrell chest, cyanosis, tripod position, use of accessory muscles Palpation: decrease movement Percussion: Hyperresonant or dull if consolidation Auscultation: Crackels, wheezes, distant breath sounds

Asthma (No exacerbation)

Inspection: Normal Palpation: Normal Percussion: Normal Auscultation: Normal

Asthma (Exacerbation)

Inspection: Prolonged expiration, tripod position, pursed lips Palpation: decrease movement Percussion: Hyperresonance Auscultation: Wheeses, decrease breath sounds ominous sign (severely diminished air movement)

Pulmonary fibrosis

Inspection: Tachypnea Palpation: Decrease Movement Percussion: Normal Auscultation: Crackles or sounds like Velcro being pulled apart

Pulmonary edema

Inspection: Tachypnea, labored respirations, cyanosis, blood-tinged sputum Palpation: Decrease movement of normal movement Percussion: Dull or normal depending on amount of fluid. Auscultation: Fine or coarse crackles

Ventilation

Involves inspiration and expiration. Air moves in and out of the lungs because intrathoracic pressure changes in relation to pressure at the airway opening.

Mechanical Receptors

Located in the lungs, upper airways, chest wall, and diaphragm. Stimulated by physiologic factors, such as irritants, muscle stretching, and alveolar wall distortion.

Role-relationship

Loss of roles at work or home, exposure to respiratory toxins at work

TLC (total lung capacity)

Maximum amount of air lungs can conain

VC (Vital capacity)

Maximum amount of air that can be exhaled after maximum inspiration

Stridor

Partial obstruction of trachea or larynx

Pleural friction rub

Pleurisy

When assessing a patient's lungs, the nurse recalls that the left lung: a.Consists of two lobes. b.Is divided by the horizontal fissure. c.Primarily consists of an upper lobe on the posterior chest. d.Is shorter than the right lung because of the underlying stomach.

The left lung has two lobes, and the right lung has three lobes. The right lung is shorter than the left lung because of the underlying liver. The left lung is narrower than the right lung because the heart bulges to the left. The posterior chest is almost all lower lobes.

Resistance

Refers to any impediment to airflow during inspiration and expiration. Affected by changes in diameter of the airways.

Oxygenation

Refers to the process of obtaining 02 from the atmospheric air and making it available to the organs and tissues of the body.

Coarse crackles

Series of long-duration, discontinuous, low-pitched sounds caused by air passing through airway intermittently occluded by mucus, unstable bronchial wall. Evident on inspiration and, at times, expiration. Similar sound to blowing through straw under water. Increase in bubbling quality with more fluid.

Sexuality-reproductive

Sexual activity altered by respiratory symptoms

Sleep-rest

Sleep apnea. awakening with dyspnea, wheezing or cough. night sweats

Elastic recoil

Tendency for the lungs to return to their normal size after being stretched or expanded.

How often should a nurse assess the skin and nares of the patient with a nasal cannula?

The nurse should assess the client's nares and ears for skin breakdown every 6 hours.

Health perception-health management

Tobacco use history, gradual changes in health status, family history of lung disease, sputum production, mo immunizations for influenza or pneumococcal pneumonia received, travel to developing countries

Fremitus

Vibration of the chest wall produced by vocalization. Can be felt by placing the palmar surface of the hands against the patient's chest witht he fingers hyperextended.

FRC (Functional residual capacity)

Volume of air in lungs after normal exhalation

Dyspnea

When SOB occurs, neck and shoulders muscles can assist the effort.

a 48-year-old client doesn't smoke cigarettes yet is demonstrating signs of lung irritation. Which of the following questions could help with the assessment of this client? a. Do you smoke or inhale marijuana or other herbal products? b. Have you had allergy testing? c. Have you received a flu or pneumonia vaccination? d. Have you tried to stop smoking?

a. Do you smoke or inhale marijuana or other herbal products?

The position of a conscious client during suctioning is: a. Fowler's b. Supine position c. Side-lying d. Prone

a. Fowler's Position a conscious person who has a functional gag reflex in the semi fowler's position with the head turned to one side for oral suctioning or with the neck hyper extended for nasal suctioning. If the client is unconscious place the patient a lateral position facing you.

During a physical assessment, the nurse documents eupnea on the client's medical record. What does this finding suggest? a. Normal respirations b. Slow respirations c. Irregular respirations d. Rapid respirations

a. Normal respirations

After inspecting a client's thorax, the nurse writes "AP:T 1:2, bilateral symmetrical movements, sternum midline, respiratory rate 16 and regular." What do these findings suggest? a. Nothing. These findings are normal. b. The client has pneumonia. c. The client has a respiratory illness. d. The client has allergies.

a. Nothing. These findings are normal.

23. The accumulation of fluids in the pleural space is called: a. Pleural effusion b. Hemothorax c. Hydrothorax d. Pyothorax

a. Pleural effusion

A client with a strained trapezius muscle complains of having occasional shortness of breath. What might be the reason for this symptom? a. The strained muscle is an accessory muscle of respiration. b. The diaphragm muscle is also injured. c. There is an undiagnosed heart problem. d. There is a blood clot in his lung.

a. The strained muscle is an accessory muscle of respiration.

Prior to listening to a client's lung sounds, the nurse palpates the sternum and feels a horizontal bump on the bone. What does this finding suggest to the nurse? a. This is the angle of Louis. b. The manubrium is damaged. c. The costal angle is greater than normal. d. The xiphoid process is misshaped.

a. This is the angle of Louis.

A 57-year-old client tells the nurse, "I need two to three pillows to sleep." How should this information be documented? a. Two to three pillow orthopnea b. Dyspnea on excursion c. Resting apnea d. Dyspnea at rest

a. Two to three pillow orthopnea

The nurse is planning to assess the apex of a client's lungs. Which area of the body will the nurse be assessing? a. Left of the sternum, third intercostal space b. Above the clavicles c. Below the scapula d. Right of the sternum, sixth intercostal space

b. Above the clavicles The apex of each lung is slightly superior to the inner third of the clavicle.

While changing the tapes on a tracheostomy tube, the male client coughs and the tube is dislodged. The initial nursing action is to: a. Call the physician to reinsert the tube. b. Grasp the retention sutures to spread the opening. c. Call the respiratory therapy department to reinsert the tracheotomy. d. Cover the tracheostomy site with a sterile dressing to prevent infection.

b. Grasp the retention sutures to spread the opening.

The nurse is assessing the client's lung bases posteriorly. At which area can the nurse assess this portion of the lung? a. Right anterior axillary line b. Scapular line c. Midsternal line d. Left midclavicular line

b. Scapular line

In planning a patient education session, the nurse sees one area of focus for Healthy People 2010 is chronic obstructive pulmonary disease (COPD). Which of the following information should the nurse include in the education session to address this focus area? a. Screening for environmental triggers b. Smoking cessation c. Develop action plans d. Identify those at risk

b. Smoking cessation

The mother of a four-year-old child tells the nurse, "I think there's something wrong with him; his chest is round like a ball." Which of the following would be an appropriate response for the nurse to make to the mother? a. I see what you mean. That seems odd. b. The chest of a child appears round and is normal. c. I wouldn't worry about that. d. Did you tell the doctor about this?

b. The chest of a child appears round and is normal.

The client tells the nurse he sometimes coughs up "thick yellow mucous." What does this information suggest to the nurse? a. He might have an allergy. b. He might have a fungal infection. c. He might have episodic lung infections. d. He might have tuberculosis.

c. He might have episodic lung infections Rationale: The color and odor of any mucus is associated with specific diseases or problems. Green or yellow mucus often signals a lung infection.

A seven-month-pregnant female is sitting quietly in the waiting room, and her respiratory rate is 20 and shallow. What does this finding suggest to the nurse? a. She has a history of smoking. b. She is using accessory muscles to breathe. b. She is in pending respiratory failure. c. Nothing. This is normal.

c. Nothing. This is normal.

While palpating the posterior thorax of a client, the nurse notes increased fremitus. What does this finding suggest to the nurse? a. The client needs to speak up. b. The client has a thick chest wall. c. The client could either have fluid in the lungs or have an infection. d. Nothing. This is a normal finding.

c. The client could either have fluid in the lungs or have an infection.

What is atelectasis?

collapse of the alveoli in the lung prevents normal exchange of O2 and co2 hypoventilation occurs

The most important action the nurse should do before and after suctioning a client is: a. Placing the client in a supine position b. Making sure that suctioning takes only 10-15 seconds c. Evaluating for clear breath sounds d. Hyperventilating the client with 100% oxygen

d. Hyperventilating the client with 100% oxygen

After examining a 75-year-old male client, the nurse writes down "barrel chest." What does this finding suggest? a. The client has a history of smoking. b. The client has osteoporosis. c. The client has long-standing respiratory disease. d. This is a change associated with aging.

d. This is a change associated with aging.

The nurse sees that the client will breathe deeply and then stop breathing for a short while. Which of the following does this observation suggest? a. This client is hyperventilating. b. This client is in a diabetic coma. c. This client has pneumonia. d. This is seen in aging people, people with heart failure, and people who have suffered brain damage.

d. This is seen in aging people, people with heart failure, and people who have suffered brain damage.

cognitive-perceptual

decreased cognitive function with restlessness, irritability. Chest pain or pain with breathing.

Adventitious breath sounds

fine crackles, coarse crackles, wheezes, stridor, and pleural friction rub

Fine crackles

interstitial edema

Increased tactile fremitus

lung consolidation with fluid or exudate

Place the organs in order of the pathway of air inspired through the nose.

nasal cavity, nasopharynx, oropharynx, laryngopharynx, epiglottis, larynx, glottis, trachea, carina, mainstem bronchi, segmental bronchi, bronchioles, alveolar duct

Fine Crackles

series of short duration, discontinuous, high-pitched sounds equalization of gas pressure when colapsed alveoli or terminal bronchioles suddenly snap open. Similar sound to that made by rolling hair between fingers.

Vt (tidal volume)

volume of air inhaled and exhaled with each breath

Describe the clinical signs of RIGHT sided heart failure.

weight gain distended neck veins hepatomegaly and splenomegaly dependent peripheral edema


Conjuntos de estudio relacionados

Chapter 15: Oligopoly and Antitrust Policy

View Set

chap 39: oxygenation/respiratory

View Set

Patient Care - Patient interactions, patient and medical record management

View Set

ARTH- 452E The Skyscraper Exam #2

View Set

JUST 4640: CH 9 - Identification Procedures (Assessment)

View Set

Quality Management Strategies Test 2-4 🟡 Review

View Set